Закон ома для участка цепи формула и определение примеры расчета: Закон Ома для участка цепи и полной цепи: формулы и объяснение

Содержание

Закон Ома для участка цепи и полной цепи: формулы и объяснение

Для электрика и электронщика одним из основных законов является Закон Ома. Каждый день работа ставит перед специалистом новые задачи, и зачастую нужно подобрать замену сгоревшему резистору или группе элементов. Электрику часто приходится менять кабеля, чтобы выбрать правильный нужно «прикинуть» ток в нагрузке, так приходится использовать простейшие физические законы и соотношения в повседневной жизни. Значение Закона Ома в электротехники колоссально, к слову большинство дипломных работ электротехнических специальностей рассчитываются на 70-90% по одной формуле.

Историческая справка

Год открытия Закон Ома  — 1826 немецким ученым Георгом Омом. Он эмпирически определил и описал закон о соотношении силы тока, напряжения и типа проводника. Позже выяснилось, что третья составляющая – это не что иное, как сопротивление. Впоследствии этот закон назвали в честь открывателя, но законом дело не ограничилось, его фамилией и назвали физическую величину, как дань уважения его работам.

Величина, в которой измеряют сопротивление, названа в честь Георга Ома. Например, резисторы имеют две основные характеристики: мощность в ваттах и сопротивление – единица измерения в Омах, килоомах, мегаомах и т.д.

Закон Ома для участка цепи

Для описания электрической цепи не содержащего ЭДС можно использовать закон Ома для участка цепи. Это наиболее простая форма записи. Он выглядит так:

I=U/R

Где I – это ток, измеряется в Амперах, U – напряжение в вольтах, R – сопротивление в Омах.

Такая формула нам говорит, что ток прямопропорционален напряжению и обратнопропорционален сопротивлению – это точная формулировка Закона Ома. Физический смысл этой формулы – это описать зависимость тока через участок цепи при известном его сопротивлении и напряжении.

Внимание! Эта формула справедлива для постоянного тока, для переменного тока она имеет небольшие отличия, к этому вернемся позже.

Кроме соотношения электрических величин данная форма нам говорит о том, что график зависимости тока от напряжения в сопротивлении линеен и выполняется уравнение функции:

f(x) = ky или f(u) = IR или f(u)=(1/R)*I

Закон Ома для участка цепи применяют для расчетов сопротивления резистора на участке схемы или для определения тока через него при известном напряжении и сопротивлении.

Например, у нас есть резистор R сопротивлением в 6 Ом, к его выводам приложено напряжение 12 В. Необходимо узнать, какой ток будет протекать через него. Рассчитаем:

I=12 В/6 Ом=2 А

Идеальный проводник не имеет сопротивления, однако из-за структуры молекул вещества, из которого он состоит, любое проводящее тело обладает сопротивлением. Например, это стало причиной перехода с алюминиевых проводов на медные в домашних электросетях. Удельное сопротивление меди (Ом на 1 метр длины) меньше чем алюминия. Соответственно медные провода меньше греются, выдерживают большие токи, значит можно использовать провод меньшего сечения.

Еще один пример — спирали нагревательных приборов и резисторов обладают большим удельным сопротивлением, т.к. изготавливаются из разных высокоомных металлов, типа нихрома, кантала и пр. Когда носители заряда движутся через проводник, они сталкиваются с частицами в кристаллической решетке, вследствие этого выделяется энергия в виде тепла и проводник нагревается. Чем больше ток – тем больше столкновений – тем больше нагрев.

Чтобы снизить нагрев проводник нужно либо укоротить, либо увеличить его толщину (площадь поперечного сечения). Эту информацию можно записать в виде формулы:

Rпровод=ρ(L/S)

Где ρ – удельное сопротивление в Ом*мм2/м, L – длина в м, S – площадь поперечного сечения.

Закон Ома для параллельной и последовательной цепи

В зависимости от типа соединения наблюдается разный характер протекания тока и распределения напряжений. Для участка цепи последовательного соединения элементов напряжение, ток и сопротивление находятся по формуле:

I=I1=I2

U=U1+U2

R=R1+R2

Это значит, что в цепи из произвольного количества последовательно соединенных элементов протекает один и тот же ток. При этом напряжение, приложенное ко всем элементам (сумма падений напряжения), равно выходному напряжению источника питания.

К каждому элементу в отдельности приложена своя величина напряжений и зависит от силы тока и сопротивления конкретного:

Uэл=I*Rэлемента

Сопротивление участка цепи для параллельно соединённых элементов рассчитывается по формуле:

I=I1+I2

U=U1=U2

1/R=1/R1+1/R2

Для смешанного соединения нужно приводить цепь к эквивалентному виду. Например, если один резистор соединен с двумя параллельно соединенными резисторами – то сперва посчитайте сопротивление параллельно соединенных. Вы получите общее сопротивление двух резисторов и вам остаётся сложить его с третьим, который с ними соединен последовательно.

Закон Ома для полной цепи

Полная цепь предполагает наличие источника питания. Идеальный источник питания – это прибор, который имеет единственную характеристику:

  • напряжение, если это источник ЭДС;
  • силу тока, если это источник тока;

Такой источник питания способен выдать любую мощность при неизменных выходных параметрах. В реальном же источнике питания есть еще и такие параметры как мощность и внутреннее сопротивление. По сути, внутреннее сопротивление – это мнимый резистор, установленный последовательно с источником ЭДС.

Формула Закона Ома для полной цепи выглядит похоже, но добавляется внутренне сопротивление ИП. Для полной цепи записывается формулой:

I=ε/(R+r)

Где ε – ЭДС в Вольтах, R – сопротивление нагрузки, r – внутреннее сопротивление источника питания.

На практике внутреннее сопротивление является долями Ома, а для гальванических источников оно существенно возрастает. Вы это наблюдали, когда на двух батарейках (новой и севшей) одинаковое напряжение, но одна выдает нужный ток и работает исправно, а вторая не работает, т.к. проседает при малейшей нагрузке.

Закон Ома в дифференциальной и интегральной форме

Для однородного участка цепи приведенные выше формулы справедливы, для неоднородного проводника необходимо его разбить на максимально короткие отрезки, чтобы изменения его размеров были минимизированы в пределах этого отрезка.

Это называется Закон Ома в дифференциальной форме.

Иначе говоря: плотность тока прямо пропорциональной напряжённости и удельной проводимости для бесконечно малого участка проводника.

В интегральной форме:

Закон Ома для переменного тока

При расчете цепей переменного тока вместо понятия сопротивления вводят понятие «импеданс». Импеданс обозначают буквой Z, в него входит активное сопротивление нагрузки Ra

и реактивное сопротивление X (или Rr). Это связано с формой синусоидального тока (и токов любых других форм) и параметрами индуктивных элементов, а также законов коммутации:

  1. Ток в цепи с индуктивностью не может измениться мгновенно.
  2. Напряжение в цепи с ёмкостью не может измениться мгновенно.

Таким образом, ток начинает отставать или опережать напряжение, и полная мощность разделяется на активную и реактивную.

U=I*Z

XL и XC – это реактивные составляющие нагрузки.

В связи с этим вводится величина cosФ:

Здесь – Q – реактивная мощность, обусловленная переменным током и индуктивно-емкостными составляющими, P – активная мощность (выделяется на активных составляющих), S – полная мощность, cosФ – коэффициент мощности.

Возможно, вы заметили, что формула и её представление пересекается с теоремой Пифагора. Это действительно так и угол Ф зависит от того, насколько велика реактивная составляющая нагрузки – чем её больше, тем он больше. На практике это приводит к тому, что реально протекающий в сети ток больше чем тот, что учитывается бытовым счетчиком, предприятия же платят за полную мощность.

При этом сопротивление представляют в комплексной форме:

Здесь j – это мнимая единица, что характерно для комплексного вида уравнений. Реже обозначается как i, но в электротехнике также обозначается и действующее значение переменного тока, поэтому, чтобы не путаться, лучше использовать j.

Мнимая единица равняется √-1. Логично, что нет такого числа при возведении в квадрат, которого может получиться отрицательный результат «-1».

Как запомнить закон Ома

Чтобы запомнить Закон Ома – можно заучить формулировку простыми словами типа:

Чем больше напряжение – тем больше ток, чем больше сопротивление – тем меньше ток.

Или воспользоваться мнемоническими картинками и правилами. Первая это представление закона Ома в виде пирамиды – кратко и понятно.

Мнемоническое правило – это упрощенный вид какого-либо понятия, для простого и легкого его понимания и изучения. Может быть либо в словесной форме, либо в графической. Чтобы правильно найти нужную формулу – закройте пальцем искомую величину и получите ответ в виде произведения или частного. Вот как это работает:

Вторая – это карикатурное представление. Здесь показано: чем больше старается Ом, тем труднее проходит Ампер, а чем больше Вольт – тем легче проходит Ампер.

Напоследок рекомендуем просмотреть полезное видео, в котором простыми словами объясняется Закон Ома и его применение:

Закон Ома – один из основополагающих в электротехнике, без его знания невозможна бОльшая часть расчетов. И в повседневной работе часто приходится переводить амперы в киловатты или по сопротивлению определять ток. Совершенно не обязательно понимать его вывод и происхождение всех величин – но конечные формулы обязательны к освоению. В заключении хочется отметить, что есть старая шуточная пословица у электриков: «Не знаешь Ома – сиди дома». И если в каждой шутке есть доля правды, то здесь эта доля правды – 100%. Изучайте теоретические основы, если хотите стать профессионалом на практике, а в этом вам помогут другие статьи из нашего сайта.

Закон Ома для «чайников»: понятие, формула, объяснение

Закон Ома для участка цепи: сила тока I на участке электрической цепи прямо пропорциональна напряжению U на концах участка и обратно пропорциональна его сопротивлению R.

Формула закона: I =. Отсюда запишем формулыU = IR и R = .

Рис.1. Участок цепи Рис.2. Полная цепь

Закон Ома для полной цепи: сила тока I полной электрической цепи равнаЭДС (электродвижущей силе) источника тока Е , деленной на полное сопротивление цепи (R + r). Полное сопротивление цепи равно сумме сопротивлений внешней цепи R и внутреннего r источника тока.Формула закона I =
. На рис. 1 и 2 приведены схемы электрических цепей.

3. Последовательное и параллельное соединение проводников

Проводники в электрических цепях могут соединяться последовательно и параллельно . Смешанное соединение сочетает оба эти соединения.

Сопротивление,при включении которого вместо всех других проводников, находящихся между двумя точками цепи, ток и напряжение остаются неизменными, называют эквивалентным сопротивлением этих проводников.

Последовательное соединение

Последовательным называется соединение, при котором каждый проводник соединяется только с одним предыдущим и одним последующим проводниками.

Как следует из первого правила Кирхгофа , при последовательном соединении проводников сила электрического тока, протекающего по всем проводникам, одинакова (на основании закона сохранения заряда).

1. При последовательном соединении проводников (рис. 1) сила тока во всех проводниках одинакова: I 1 = I 2 = I 3 = I

Рис. 1.Последовательное соединение двух проводников.

2. Согласно закону Ома, напряженияU 1 иU 2 на проводниках равны U 1 = IR 1 , U 2 = IR 2 , U 3 = IR 3 .

Напряжение при последовательном соединении проводников равно сумме напряжений на отдельных участках (проводниках) электрической цепи.

U = U 1 + U 2 + U 3

Позакону Ома, напряжения U 1, U 2 на проводниках равныU 1 = IR 1 , U 2 = IR 2 , В соответствии вторым правилом Кирхгофа напряжение на всем участке:

U = U 1 + U 2 = IR 1 + IR 2 = I(R 1 + R 2 )= I·R. Получаем: R = R 1 + R 2

Общее напряжение U на проводниках равно сумме напряжений U 1 , U 2 , U 3 равно: U = U 1 + U 2 + U 3 = I · (R 1 + R 2 + R 3 ) = IR

где R ЭКВ эквивалентное сопротивление всей цепи. Отсюда: R ЭКВ = R 1 + R 2 + R 3

При последовательном соединении эквивалентное сопротивление цепи равно сумме сопротивлений отдельных участков цепи: R ЭКВ = R 1 + R 2 + R 3 +…

Этот результат справедлив для любого числа последовательно соединенных проводников.

Из закона Омаследует: при равенстве сил тока при последовательном соединении:

I = , I = . Отсюда = или =, т. е. напряжения на отдельных участках цепи прямо пропорциональны сопротивлениям участков.

При последовательном соединении n одинаковых проводников общее напряжение равно произведению напряжению одного U 1 на их количество n :

U ПОСЛЕД = n · U 1 . Аналогично для сопротивлений: R ПОСЛЕД = n · R 1

При размыкании цепи одного из последовательно соединенных потребителей ток исчезает во всей цепи, поэтому последовательное соединение на практике не всегда удобно.

Вся прикладная электротехника базируется на одном догмате – это закон Ома для участка цепи. Без понимания принципа этого закона невозможно приступать к практике, поскольку это приводит к многочисленным ошибкам. Имеет смысл освежить эти знания, в статье мы напомним трактовку закона, составленного Омом, для однородного и неоднородного участка и полной цепи.

Классическая формулировка

Этот простой вариант трактовки, известный нам со школы.


Формула в интегральной форме будет иметь следующий вид:


То есть, поднимая напряжение, мы тем самым увеличиваем ток. В то время, как увеличение такого параметра, как «R», ведет к снижению «I». Естественно, что на рисунке сопротивление цепи показано одним элементом, хотя это может быть последовательное, параллельное (вплоть до произвольного)соединение нескольких проводников.

В дифференциальной форме закон мы приводить не будем, поскольку в таком виде он применяется, как правило, только в физике.

Принятые единицы измерения

Необходимо учитывать, что все расчеты должны проводиться в следующих единицах измерения:

  • напряжение – в вольтах;
  • ток в амперах
  • сопротивление в омах.

Если вам встречаются другие величины, то их необходимо будет перевести к общепринятым.

Формулировка для полной цепи

Трактовка для полной цепи будет несколько иной, чем для участка, поскольку в законе, составленном Омом, еще учитывает параметр «r», это сопротивление источника ЭДС. На рисунке ниже проиллюстрирована подобная схема.


Учитывая «r» ЭДС, формула предстанет в следующем виде:


Заметим, если «R» сделать равным 0, то появляется возможность рассчитать «I», возникающий во время короткого замыкания.

Напряжение будет меньше ЭДС, определить его можно по формуле:


Собственно, падение напряжения характеризуется параметром «I*r». Это свойство характерно многим гальваническим источникам питания.

Неоднородный участок цепи постоянного тока

Под таким типом подразумевается участок, где помимо электрического заряда производится воздействие других сил. Изображение такого участка показано на рисунке ниже.


Формула для такого участка (обобщенный закон) будет иметь следующий вид:


Переменный ток

Если в схема, подключенная к переменному току снабжена емкостью и/или индуктивностью (катушкой), расчет производится с учетом величин их реактивных сопротивлений. Упрощенный вид закона будет выглядеть следующим образом:

Где «Z» представляет собой импеданс, это комплексная величина, состоящая из активного (R) и пассивного (Х) сопротивлений.

Практическое использование

Видео: Закон Ома для участка цепи – практика расчета цепей.

Собственно, к любому участку цепи можно применить этот закон. Пример приведен на рисунке.


Используя такой план, можно вычислить все необходимые характеристики для неразветвленного участка. Рассмотрим более детальные примеры.
Находим силу тока
Рассмотрим теперь более определенный пример, допустим, возникла необходимость узнать ток, протекающий через лампу накаливания. Условия:

  • Напряжение – 220 В;
  • R нити накала – 500 Ом.

Решение задачи будет выглядеть следующим образом: 220В/500Ом=0,44 А.

Рассмотрим еще одну задачу со следующими условиями:

В этом случае, в первую очередь, потребуется выполнить преобразование: 0,2 МОм = 200000 Ом,после чего можно приступать к решению: 400 В/200000 Ом=0,002 А (2 мА).
Вычисление напряжения
Для решения мы также воспользуемся законом, составленным Омом. Итак задача:

Преобразуем исходные данные:

  • 20 кОм = 20000 Ом;
  • 10 мА=0,01 А.

Решение: 20000 Ом х 0,01 А = 200 В.

Незабываем преобразовывать значения, поскольку довольно часто ток может быть указан в миллиамперах.

Сопротивление.

Несмотря на то, что общий вид способа для расчета параметра «R» напоминает нахождение значения «I», между этими вариантами существуют принципиальные различия. Если ток может меняться в зависимости от двух других параметров, то R (на практике) имеет постоянное значение. То есть по своей сути оно представляется в виде неизменной константы.

Если через два разных участка проходит одинаковый ток (I), в то время как приложенное напряжение (U) различается, то, опираясь на рассматриваемый нами закон, можно с уверенностью сказать, что там где низкое напряжение «R» будет наименьшим.

Рассмотрим случай когда разные токи и одинаковое напряжение на несвязанных между собой участках. Согласно закону, составленному Омом, большая сила тока будет характерна небольшому параметру «R».

Рассмотрим несколько примеров.

Допустим, имеется цепь, к которой подведено напряжение U=50 В, а потребляемый ток I=100 мА. Чтобы найти недостающий параметр, следует 50 В / 0,1 А (100 мА), в итоге решением будет – 500 Ом.

Вольтамперная характеристика позволяет наглядно продемонстрировать пропорциональную (линейную) зависимость закона. На рисунке ниже составлен график для участка с сопротивлением равным одному Ому (почти как математическое представление закона Ома).

Изображение вольт-амперной характеристики, где R=1 Ом


Изображение вольт-амперной характеристики

Вертикальная ось графика отображает ток I (A), горизонтальная – напряжение U(В). Сам график представлен в виде прямой линии, которая наглядно отображает зависимость от сопротивления, которое остается неизменным. Например, при 12 В и 12 А «R» будет равно одному Ому (12 В/12 А).

Обратите внимание, что на приведенной вольтамперной характеристике отображены только положительные значения. Это указывает, что цепь рассчитана на протекание тока в одном направлении. Там где допускается обратное направление, график будет продолжен на отрицательные значения.

Заметим, что оборудование, вольт-амперная характеристика которого отображена в виде прямой линии, именуется – линейным. Этот же термин используется для обозначения и других параметров.

Помимо линейного оборудования, есть различные приборы, параметр «R» которых может меняться в зависимости от силы тока или приложенного напряжения. В этом случая для расчета зависимости нельзя использовать закон Ома. Оборудование такого типа называется нелинейным, соответственно, его вольт-амперные характеристики не будут отображены в виде прямых линий.

Вывод

Как уже упоминалось в начале статьи, вся прикладная электротехника базируется на законе, составленном Омом. Незнание этого базового догмата может привести к неправильному расчету, который, в свою очередь, станет причиной аварии.

Подготовка электриков как специалистов начинается с изучения теоретических основ электротехники. И первое, что они должны запомнить – это закон составленный Омом, поскольку на его основе производятся практически все расчеты параметров электрических цепей различного назначения.

Понимание основного закона электротехники поможет лучше разбираться в работе электрооборудования и его основных компонентов. Это положительно отразится на техническом обслуживании в процессе эксплуатации.

Самостоятельная проверка, разработка, а также опытное изучение узлов оборудования – все это существенно упрощается, если использовать закон Ома для участка цепи. При этом не требуется проводить всех измерений, достаточно снять некоторые параметры и, проведя несложные расчеты, получить необходимые значения.

Если увеличить в несколько раз напряжение, действующее в электрической цепи, то ток в этой цепи увеличится во столько же раз. А если увеличить в несколько раз сопротивление цепи, то ток во столько же раз уменьшится. Подобно этому водяной поток в трубе тем больше, чем сильнее давление и чем меньше сопротивление, которое оказывает труба движению воды.

Чтобы выразить закон Ома математически наиболее просто, считают, что сопротивление проводника, в котором при напряжении 1 В проходит ток 1 А, равно 1 Ом.

Ток в амперах можно всегда определить, если разделить напряжение в вольтах на сопротивление в омах. Поэтому закон Ома для участка цепи записывается следующей формулой:

Расчеты, выполняемые с помощью закона Ома для участка цепи, будут правильны в том случае, когда напряжение выражено в вольтах, сопротивление в омах и ток в амперах. Если используются кратные единицы измерений этих величин (например, миллиампер, милливольт, мегаом и т. д.), то их следует перевести соответственно в амперы, вольты и омы. Чтобы подчеркнуть это, иногда формулу закона Ома для участка цепи пишут так:

ампер = вольт/ом

Можно также рассчитывать ток в миллиамперах и микроамперах, при этом напряжение должно быть выражено в вольтах, а сопротивление – в килоомах и мегаомах соответственно.

Закон Ома справедлив для любого участка цепи. Если требуется определить ток в данном участке цепи, то необходимо напряжение, действующее на этом участке (рис. 1), разделить на сопротивление именно этого участка.

Рис 1. Применение закона Ома для участка цепи

Приведем пример расчета тока по закону Ома . Пусть требуется определить ток в лампе, имеющей сопротивление 2,5 Ом, если напряжение, приложенное к лампе, составляет 5 В. Разделив 5 В на 2,5 Ом, получим значение тока, равное 2 А. Во втором примере определим ток, который будет протекать под действием напряжения 500 В в цепи, сопротивление которой равно 0,5 МОм. Для этого выразим сопротивление в омах. Разделив 500 В на 500 000 Ом, найдем значение тока в цепи, которое равно 0,001 А или 1 мА.

Часто, зная ток и сопротивление, определяют с помощью закона Ома напряжение. Запишем формулу для определения напряжения

Из этой формулы видно, что напряжение на концах данного участка цепи прямо пропорционально току и сопротивлению . Смысл этой зависимости понять нетрудно. Если не изменять сопротивление участка цепи, то увеличить ток можно только путем увеличения напряжения. Значит при постоянном сопротивлении большему току соответствует большее напряжение. Если же надо получить один и тот же ток при различных сопротивлениях, то при большем сопротивлении должно быть соответственно большее напряжение.

Напряжение на участке цепи часто называют падением напряжения . Это нередко приводит к недоразумению. Многие думают, что падение напряжения есть какое-то потерянное ненужное напряжение. В действительности же понятия напряжение и падение напряжения равнозначны.

Расчет напряжения с помощью закона Ома можно показать на следующем примере. Пусть через участок цепи с сопротивлением 10 кОм проходит ток 5 мА и требуется определить напряжение на этом участке.

Умножив I = 0,005 А на R -10000 Ом, получим напряжение,равное 50 В. Можно было бы получить тот же результат, умножив 5 мА на 10 кОм: U = 50 В

В электронных устройствах ток обычно выражается в миллиамперах, а сопротивление – в килоомах. Поэтому удобно в расчетах по закону Ома применять именно эти единицы измерений.

По закону Ома рассчитывается также сопротивление, если известно напряжение и ток. Формула для этого случая пишется следующим образом: R = U/I.

Сопротивление всегда представляет собой отношение напряжения к току. Если напряжение увеличить или уменьшить в несколько раз, то ток увеличится или уменьшится в такое же число раз. Отношение напряжения к току, равное сопротивлению, остается неизменным.

Не следует понимать формулу для определения сопротивления в том смысле, что сопротивление данного проводника зависит оттока и напряжения. Известно, что оно зависит от длины, площади сечения и материала проводника. По внешнему виду формула для определения сопротивления напоминает формулу для расчета тока, но между ними имеется принципиальная разница. Ток в данном участке цепи действительно зависит от напряжения и сопротивления и изменяется при их изменении. А сопротивление данного участка цепи является величиной постоянной, не зависящей от изменения напряжения и тока, но равной отношению этих величин.

Когда один и тот же ток проходит в двух участках цепи, а напряжения, приложенные к ним, различны, то ясно, что участок, к которому приложено большее напряжение, имеет соответственно большее сопротивление. А если под действием одного и того же напряжения в двух разных участках цепи проходит различный ток, то меньший ток всегда будет на том участке, который имеет большее сопротивление. Все это вытекает из основной формулировки закона Ома для участка цепи, т. е. из того, что ток тем больше, чем больше напряжение и чем меньше сопротивление.

Расчет сопротивления с помощью закона Ома для участка цепи покажем на следующем примере. Пусть требуется найти сопротивление участка, через который при напряжении 40 В проходит ток 50 мА. Выразив ток в амперах, получим I = 0,05 А. Разделим 40 на 0,05 и найдем, что сопротивление составляет 800 Ом.

Закон Ома можно наглядно представить в виде так называемой вольт-амперной характеристики . Как известно, прямая пропорциональная зависимость между двумя величинами представляет собой прямую линию, проходящую через начало координат. Такую зависимость принято называть линейной .

Для электрика и электронщика одним из основных законов является Закон Ома. Каждый день работа ставит перед специалистом новые задачи, и зачастую нужно подобрать замену сгоревшему резистору или группе элементов. Электрику часто приходится менять кабеля, чтобы выбрать правильный нужно «прикинуть» ток в нагрузке, так приходится использовать простейшие физические законы и соотношения в повседневной жизни. Значение Закона Ома в электротехники колоссально, к слову большинство дипломных работ электротехнических специальностей рассчитываются на 70-90% по одной формуле.

Историческая справка

Год открытия Закон Ома — 1826 немецким ученым Георгом Омом. Он эмпирически определил и описал закон о соотношении силы тока, напряжения и типа проводника. Позже выяснилось, что третья составляющая – это не что иное, как сопротивление. Впоследствии этот закон назвали в честь открывателя, но законом дело не ограничилось, его фамилией и назвали физическую величину, как дань уважения его работам.

Величина, в которой измеряют сопротивление, названа в честь Георга Ома. Например, резисторы имеют две основные характеристики: мощность в ваттах и сопротивление – единица измерения в Омах, килоомах, мегаомах и т.д.

Закон Ома для участка цепи

Для описания электрической цепи не содержащего ЭДС можно использовать закон Ома для участка цепи. Это наиболее простая форма записи. Он выглядит так:

Где I – это ток, измеряется в Амперах, U – напряжение в вольтах, R – сопротивление в Омах.

Такая формула нам говорит, что ток прямопропорционален напряжению и обратнопропорционален сопротивлению – это точная формулировка Закона Ома. Физический смысл этой формулы – это описать зависимость тока через участок цепи при известном его сопротивлении и напряжении.

Внимание! Эта формула справедлива для постоянного тока, для переменного тока она имеет небольшие отличия, к этому вернемся позже.

Кроме соотношения электрических величин данная форма нам говорит о том, что график зависимости тока от напряжения в сопротивлении линеен и выполняется уравнение функции:

f(x) = ky или f(u) = IR или f(u)=(1/R)*I

Закон Ома для участка цепи применяют для расчетов сопротивления резистора на участке схемы или для определения тока через него при известном напряжении и сопротивлении. Например, у нас есть резистор R сопротивлением в 6 Ом, к его выводам приложено напряжение 12 В. Необходимо узнать, какой ток будет протекать через него. Рассчитаем:

I=12 В/6 Ом=2 А

Идеальный проводник не имеет сопротивления, однако из-за структуры молекул вещества, из которого он состоит, любое проводящее тело обладает сопротивлением. Например, это стало причиной перехода с алюминиевых проводов на медные в домашних электросетях. Удельное сопротивление меди (Ом на 1 метр длины) меньше чем алюминия. Соответственно медные провода меньше греются, выдерживают большие токи, значит можно использовать провод меньшего сечения.

Еще один пример — спирали нагревательных приборов и резисторов обладают большим удельным сопротивлением, т.к. изготавливаются из разных высокоомных металлов, типа нихрома, кантала и пр. Когда носители заряда движутся через проводник, они сталкиваются с частицами в кристаллической решетке, вследствие этого выделяется энергия в виде тепла и проводник нагревается. Чем больше ток – тем больше столкновений – тем больше нагрев.

Чтобы снизить нагрев проводник нужно либо укоротить, либо увеличить его толщину (площадь поперечного сечения). Эту информацию можно записать в виде формулы:

R провод =ρ(L/S)

Где ρ – удельное сопротивление в Ом*мм 2 /м, L – длина в м, S – площадь поперечного сечения.

Закон Ома для параллельной и последовательной цепи

В зависимости от типа соединения наблюдается разный характер протекания тока и распределения напряжений. Для участка цепи последовательного соединения элементов напряжение, ток и сопротивление находятся по формуле:

Это значит, что в цепи из произвольного количества последовательно соединенных элементов протекает один и тот же ток. При этом напряжение, приложенное ко всем элементам (сумма падений напряжения), равно выходному напряжению источника питания. К каждому элементу в отдельности приложена своя величина напряжений и зависит от силы тока и сопротивления конкретного:

U эл =I*R элемента

Сопротивление участка цепи для параллельно соединённых элементов рассчитывается по формуле:

1/R=1/R1+1/R2

Для смешанного соединения нужно приводить цепь к эквивалентному виду. Например, если один резистор соединен с двумя параллельно соединенными резисторами – то сперва посчитайте сопротивление параллельно соединенных. Вы получите общее сопротивление двух резисторов и вам остаётся сложить его с третьим, который с ними соединен последовательно.

Закон Ома для полной цепи

Полная цепь предполагает наличие источника питания. Идеальный источник питания – это прибор, который имеет единственную характеристику:

  • напряжение, если это источник ЭДС;
  • силу тока, если это источник тока;

Такой источник питания способен выдать любую мощность при неизменных выходных параметрах. В реальном же источнике питания есть еще и такие параметры как мощность и внутреннее сопротивление. По сути, внутреннее сопротивление – это мнимый резистор, установленный последовательно с источником ЭДС.

Формула Закона Ома для полной цепи выглядит похоже, но добавляется внутренне сопротивление ИП. Для полной цепи записывается формулой:

I=ε/(R+r)

Где ε – ЭДС в Вольтах, R – сопротивление нагрузки, r – внутреннее сопротивление источника питания.

На практике внутреннее сопротивление является долями Ома, а для гальванических источников оно существенно возрастает. Вы это наблюдали, когда на двух батарейках (новой и севшей) одинаковое напряжение, но одна выдает нужный ток и работает исправно, а вторая не работает, т.к. проседает при малейшей нагрузке.

Закон Ома в дифференциальной и интегральной форме

Для однородного участка цепи приведенные выше формулы справедливы, для неоднородного проводника необходимо его разбить на максимально короткие отрезки, чтобы изменения его размеров были минимизированы в пределах этого отрезка. Это называется Закон Ома в дифференциальной форме.

Иначе говоря: плотность тока прямо пропорциональной напряжённости и удельной проводимости для бесконечно малого участка проводника.

В интегральной форме:

Закон Ома для переменного тока

При расчете цепей переменного тока вместо понятия сопротивления вводят понятие «импеданс». Импеданс обозначают буквой Z, в него входит активное сопротивление нагрузки R a и реактивное сопротивление X (или R r). Это связано с формой синусоидального тока (и токов любых других форм) и параметрами индуктивных элементов, а также законов коммутации:

  1. Ток в цепи с индуктивностью не может измениться мгновенно.
  2. Напряжение в цепи с ёмкостью не может измениться мгновенно.

Таким образом, ток начинает отставать или опережать напряжение, и полная мощность разделяется на активную и реактивную.

X L и X C – это реактивные составляющие нагрузки.

В связи с этим вводится величина cosФ:

Здесь – Q – реактивная мощность, обусловленная переменным током и индуктивно-емкостными составляющими, P – активная мощность (выделяется на активных составляющих), S – полная мощность, cosФ – коэффициент мощности.

Возможно, вы заметили, что формула и её представление пересекается с теоремой Пифагора. Это действительно так и угол Ф зависит от того, насколько велика реактивная составляющая нагрузки – чем её больше, тем он больше. На практике это приводит к тому, что реально протекающий в сети ток больше чем тот, что учитывается бытовым счетчиком, предприятия же платят за полную мощность.

При этом сопротивление представляют в комплексной форме:

Здесь j – это мнимая единица, что характерно для комплексного вида уравнений. Реже обозначается как i, но в электротехнике также обозначается и действующее значение переменного тока, поэтому, чтобы не путаться, лучше использовать j.

Мнимая единица равняется √-1. Логично, что нет такого числа при возведении в квадрат, которого может получиться отрицательный результат «-1».

Как запомнить закон Ома

Чтобы запомнить Закон Ома – можно заучить формулировку простыми словами типа:

Чем больше напряжение – тем больше ток, чем больше сопротивление – тем меньше ток.

Или воспользоваться мнемоническими картинками и правилами. Первая это представление закона Ома в виде пирамиды – кратко и понятно.

Мнемоническое правило – это упрощенный вид какого-либо понятия, для простого и легкого его понимания и изучения. Может быть либо в словесной форме, либо в графической. Чтобы правильно найти нужную формулу – закройте пальцем искомую величину и получите ответ в виде произведения или частного. Вот как это работает:

Вторая – это карикатурное представление. Здесь показано: чем больше старается Ом, тем труднее проходит Ампер, а чем больше Вольт – тем легче проходит Ампер.

Закон Ома – один из основополагающих в электротехнике, без его знания невозможна бОльшая часть расчетов. И в повседневной работе часто приходится переводить или по сопротивлению определять ток. Совершенно не обязательно понимать его вывод и происхождение всех величин – но конечные формулы обязательны к освоению. В заключении хочется отметить, что есть старая шуточная пословица у электриков: «Не знаешь Ома – сиди дома». И если в каждой шутке есть доля правды, то здесь эта доля правды – 100%. Изучайте теоретические основы, если хотите стать профессионалом на практике, а в этом вам помогут другие статьи из нашего сайта.

Нравится(0 ) Не нравится(0 )

Мы начинаем публикацию материалов новой рубрики “” и в сегодняшней статье речь пойдет о фундаментальных понятиях, без которых не проходит обсуждение ни одного электронного устройства или схемы. Как вы уже догадались, я имею ввиду ток, напряжение и сопротивление 😉 Кроме того, мы не обойдем стороной закон, который определяет взаимосвязь этих величин, но не буду забегать вперед, давайте двигаться постепенно.

Итак, давайте начнем с понятия напряжения .

Напряжение.

По определению напряжение – это энергия (или работа), которая затрачивается на перемещение единичного положительного заряда из точки с низким потенциалом в точку с высоким потенциалом (т. е. первая точка имеет более отрицательный потенциал по сравнению со второй). Из курса физики мы помним, что потенциал электростатического поля – это скалярная величина, равная отношению потен­циальной энергии заряда в поле к этому заряду. Давайте рассмотрим небольшой пример:

В пространстве действует постоянное электрическое поле, напряженность которого равна E . Рассмотрим две точки, расположенные на расстоянии d друг от друга. Так вот напряжение между двумя точками представляет из себя ни что иное, как разность потенциалов в этих точках:

В то же время не забываем про связь напряженности электростатического поля и разности потенциалов между двумя точками:

И в итоге получаем формулу, связывающую напряжение и напряженность:

В электронике, при рассмотрении различных схем, напряжение все-таки принято считать как разность потенциалов между точками. Соответственно, становится понятно, что напряжение в цепи – это понятие, связанное с двумя точками цепи. То есть говорить, к примеру, “напряжение в резисторе” – не совсем корректно. А если говорят о напряжении в какой-то точке, то подразумевают разность потенциалов между этой точкой и “землей” . Вот так плавно мы вышли к еще одному важнейшему понятию при изучении электроники, а именно к понятию “земля” 🙂 Так вот “землей” в электрических цепях чаще всего принято считать точку нулевого потенциала (то есть потенциал этой точки равен 0).

Давайте еще пару слов скажем о единицах, которые помогают охарактеризовать величину напряжения . Единицей измерения является Вольт (В) . Глядя на определение понятия напряжения мы можем легко понять, что для перемещения заряда величиной 1 Кулон между точками, имеющими разность потенциалов 1 Вольт , необходимо совершить работу, равную 1 Джоулю . С этим вроде бы все понятно и можно двигаться дальше 😉

А на очереди у нас еще одно понятие, а именно ток .

Ток, сила тока в цепи.

Что же такое электрический ток ?

Давайте подумаем, что будет происходить если под действие электрического поля попадут заряженные частицы, например, электроны…Рассмотрим проводник, к которому приложено определенное напряжение :

Из направления напряженности электрического поля (E ) мы можем сделать вывод о том, что title=”Rendered by QuickLaTeX.com”> (вектор напряженности всегда направлен в сторону уменьшения потенциала). На каждый электрон начинает действовать сила:

Где e – это заряд электрона.

И поскольку электрон является отрицательно заряженной частицей, то вектор силы будет направлен в сторону противоположную направлению вектора напряженности поля. Таким образом, под действием силы частицы наряду с хаотическим движением приобретают и направленное (вектор скорости V на рисунке). В результате и возникает электрический ток 🙂

Ток – это упорядоченное движение заряженных частиц под воздействием электрического поля.

Важным нюансом является то, что принято считать, что ток протекает от точки с более положительным потенциалом к точке с более отрицательным потенциалом, несмотря на то, что электрон перемещается в противоположном направлении.

Носителями заряда могут выступать не только электроны. Например, в электролитах и ионизированных газах протекание тока в первую очередь связано с перемещением ионов, которые являются положительно заряженными частицами. Соответственно, направление вектора силы, действующей на них (а заодно и вектора скорости) будет совпадать с направлением вектора E . И в этом случае противоречия не возникнет, ведь ток будет протекать именно в том направлении, в котором движутся частицы 🙂

Для того, чтобы оценить ток в цепи придумали такую величину как сила тока. Итак, сила тока (I ) – это величина, которая характеризует скорость перемещения электрического заряда в точке. Единицей измерения силы тока является Ампер . Сила тока в проводнике равна 1 Амперу , если за 1 секунду через поперечное сечение проводника проходит заряд 1 Кулон .

Мы уже рассмотрели понятия силы тока и напряжения , теперь давайте разберемся каким образом эти величины связаны. И для этого нам предстоит изучить, что же из себя представляет сопротивление проводника .

Сопротивление проводника/цепи.

Термин “сопротивление ” уже говорит сам за себя 😉

Итак, сопротивление – физическая величина, характеризующая свойства проводника препятствовать (сопротивляться ) прохождению электрического тока.

Рассмотрим медный проводник длиной l с площадью поперечного сечения, равной S :

Сопротивление проводника зависит от нескольких факторов:

Удельное сопротивление – это табличная величина.

Формула, с помощью которой можно вычислить сопротивление проводника выглядит следующим образом:

Для нашего случая будет равно 0,0175 (Ом * кв. мм / м) – удельное сопротивление меди. Пусть длина проводника составляет 0.5 м , а площадь поперечного сечения равна 0. 2 кв. мм . Тогда:

Как вы уже поняли из примера, единицей измерения сопротивления является Ом 😉

С сопротивлением проводника все ясно, настало время изучить взаимосвязь напряжения, силы тока и сопротивления цепи .

И тут на помощь нам приходит основополагающий закон всей электроники – закон Ома:

Сила тока в цепи прямо пропорциональна напряжению и обратно пропорциональна сопротивлению рассматриваемого участка цепи.

Рассмотрим простейшую электрическую цепь:

Как следует из закона Ома напряжение и сила тока в цепи связаны следующим образом:

Пусть напряжение составляет 10 В, а сопротивление цепи равно 200 Ом. Тогда сила тока в цепи вычисляется следующим образом:

Как видите, все несложно 🙂

Пожалуй на этом мы и закончим сегодняшнюю статью, спасибо за внимание и до скорых встреч! 🙂

Закон ома для участка цепи гласит. Закон Ома для участка цепи

Закон Ома.

I = U/ R

Где U – напряжение концов участка,I– сила тока, R– сопротивление проводника.

R = U / I

Эти формулы справедливы лишь когда сеть испытывает на себе одно сопротивление.

Условием движения электрических зарядов в проводнике является наличие в нем электрического поля, которое создается и поддерживается особыми устройствами, получившими название источников тока .

Основной величиной, характеризующей источник тока, является его электродвижущая сила.

Электродвижущей силой источника (сокращенно ЭДС) называется скалярная физическая величина, характеризующая работу сторонних сил, способных создавать на зажимах источника (полюсах) разность потенциалов.

Она равна работе сторонних сил по перемещению заряженной частицы с положительным единичным зарядом от одного полюса источника к другому, т.е.

В СИ ЭДС измеряется в вольтах (В), т.е. в тех же единицах, что и напряжение.

Сторонние силы источника – это силы, которые осуществляют разделение зарядов в источнике и тем самым создают на его полюсах разность потенциалов. Эти силы могут иметь различную природу, но только не электрическую (отсюда и название) – Механические силы, химическая среда в аккумуляторе; световой поток в фотоэлементах.

Направление ЭДС – это направление принудительного движения положительных зарядов внутри генератора от минуса к плюсу под действием иной, чем электрическая, природы.

Внутреннее сопротивление генератора это сопротивление конструктивных элементов внутри него.

Если электрическую цепь разделить на два участка – внешний, с сопротивлением R , и внутренний, с сопротивлением r , то ЭДС источника тока окажется равной сумме напряжений на внешнем и внутреннем участках цепи:

По закону Ома напряжение на любом участке цепи определяется величиной протекающего тока и его сопротивлением:

Так как , следовательно

, (3)

т.е. напряжение на полюсах источника при замкнутой цепи зависит от соотношения сопротивлений внутреннего и внешнего участков цепи. Если приблизительно равно U .

Электрическое сопротивление.

Свойство материала проводника препятствовать прохождению через него электрического тока называется электрическим сопротивлением.

Из закона Ома: R = U / I

За единицу электрического сопротивления принят 1Ом .

Сопротивлением 1 Ом обладает проводник, в котором при напряжении 1 В проходит ток 1 А.

Величина, обратная сопротивлению, называется электрической проводимостью :

Единицей проводимости является сименс :

Величина, обратная удельной проводимости, называется удельным сопротивлением р, т. е.

Увеличение температуры сопровождается усилением хаотического теплового движения частиц вещества, что приводит к увеличению числа столкновений электронов с ними и затрудняет упорядоченное движение электронов.

Сопротивление – резистор.

Метод узловых потенциалов.

Пример 2. 7.4.

Определить значения и направления токов в ветвях методом узловых потенциалов для цепи рис. 2.7.4, если:

Е1=108 В; Е2=90 В; Ri1=2 Ом; Ri2=1 Ом; R1=28 Ом; R2=39 Ом; R3=60 Ом.

Решение.

Определяем токи в ветвях.


Метод двух узлов.

Одним из распространенных методов расчета электрических цепей является метод двух узлов .Этот метод применяется в случае, когда в цепи всего два узла

Метод контурных токов.

Алгоритм действий таков:

По второму закону Кирхгофа, относительно контурных токов, составляем уравнения для всех независимых контуров. При записи равенства считать, что направление обхода контура, для которого составляется уравнение, совпадает с направлением контурного тока данного контура. Следует учитывать и тот факт, что в смежных ветвях, принадлежащих двум контурам, протекают два контурных тока. Падение напряжения на потребителях в таких ветвях надо брать от каждого тока в отдельности.

Произвольно задаемся направлением реальных токов всех ветвей и обозначаем их. Маркировать реальные токи надо таким образом, чтобы не путать с контурными. Для нумерации реальных токов можно использовать одиночные арабские цифры (I1, I2, I3 и т. д.).

При алгебраическом суммировании без изменения знака берется контурный ток, направление которого совпадает с принятым направлением реального тока ветви. В противном случае контурный ток умножается на минус единицу.

Пример расчёта сложной цепи методом контурных токов.

Рис. 1. Схема электрической цепи для примера расчета по методу контурных токов

Решение. Для расчета сложной цепи этим методом достаточно составить два уравнения, по числу независимых контуров. Контурные токи направляем по часовой стрелке и обозначаем I11 и I22 (см. рисунок 1).

По второму закону Кирхгофа относительно контурных токов составляем уравнения:

Решаем систему и получаем контурные токи I11 = I22 = 3 А.

Следует отметить, как положительный факт, что в методе контурных токов по сравнению с решением по законам Кирхгофа приходится решать систему уравнений меньшего порядка. Однако этот метод не позволяет сразу определять реальные токи ветвей.

Закон Ома.

Согласно закону Ома для некоторого участка цепи, сила тока на участке цепи прямо пропорциональна напряжению на концах участка и обратно пропорциональна сопротивлению.

Если увеличить в несколько раз напряжение, действующее в электрической цепи, то ток в этой цепи увеличится во столько же раз. А если увеличить в несколько раз сопротивление цепи, то ток во столько же раз уменьшится. Подобно этому водяной поток в трубе тем больше, чем сильнее давление и чем меньше сопротивление, которое оказывает труба движению воды.

Чтобы выразить закон Ома математически наиболее просто, считают, что сопротивление проводника, в котором при напряжении 1 В проходит ток 1 А, равно 1 Ом.

Ток в амперах можно всегда определить, если разделить напряжение в вольтах на сопротивление в омах. Поэтому закон Ома для участка цепи записывается следующей формулой:

Расчеты, выполняемые с помощью закона Ома для участка цепи, будут правильны в том случае, когда напряжение выражено в вольтах, сопротивление в омах и ток в амперах. Если используются кратные единицы измерений этих величин (например, миллиампер, милливольт, мегаом и т. д.), то их следует перевести соответственно в амперы, вольты и омы. Чтобы подчеркнуть это, иногда формулу закона Ома для участка цепи пишут так:

ампер = вольт/ом

Можно также рассчитывать ток в миллиамперах и микроамперах, при этом напряжение должно быть выражено в вольтах, а сопротивление – в килоомах и мегаомах соответственно.

Закон Ома справедлив для любого участка цепи. Если требуется определить ток в данном участке цепи, то необходимо напряжение, действующее на этом участке (рис. 1), разделить на сопротивление именно этого участка.

Рис 1. Применение закона Ома для участка цепи

Приведем пример расчета тока по закону Ома . Пусть требуется определить ток в лампе, имеющей сопротивление 2,5 Ом, если напряжение, приложенное к лампе, составляет 5 В. Разделив 5 В на 2,5 Ом, получим значение тока, равное 2 А. Во втором примере определим ток, который будет протекать под действием напряжения 500 В в цепи, сопротивление которой равно 0,5 МОм. Для этого выразим сопротивление в омах. Разделив 500 В на 500 000 Ом, найдем значение тока в цепи, которое равно 0,001 А или 1 мА.

Часто, зная ток и сопротивление, определяют с помощью закона Ома напряжение. Запишем формулу для определения напряжения

Из этой формулы видно, что напряжение на концах данного участка цепи прямо пропорционально току и сопротивлению . Смысл этой зависимости понять нетрудно. Если не изменять сопротивление участка цепи, то увеличить ток можно только путем увеличения напряжения. Значит при постоянном сопротивлении большему току соответствует большее напряжение. Если же надо получить один и тот же ток при различных сопротивлениях, то при большем сопротивлении должно быть соответственно большее напряжение.

Напряжение на участке цепи часто называют падением напряжения . Это нередко приводит к недоразумению. Многие думают, что падение напряжения есть какое-то потерянное ненужное напряжение. В действительности же понятия напряжение и падение напряжения равнозначны.

Расчет напряжения с помощью закона Ома можно показать на следующем примере. Пусть через участок цепи с сопротивлением 10 кОм проходит ток 5 мА и требуется определить напряжение на этом участке.

Умножив I = 0,005 А на R -10000 Ом, получим напряжение,равное 50 В. Можно было бы получить тот же результат, умножив 5 мА на 10 кОм: U = 50 В

В электронных устройствах ток обычно выражается в миллиамперах, а сопротивление – в килоомах. Поэтому удобно в расчетах по закону Ома применять именно эти единицы измерений.

По закону Ома рассчитывается также сопротивление, если известно напряжение и ток. Формула для этого случая пишется следующим образом: R = U/I.

Сопротивление всегда представляет собой отношение напряжения к току. Если напряжение увеличить или уменьшить в несколько раз, то ток увеличится или уменьшится в такое же число раз. Отношение напряжения к току, равное сопротивлению, остается неизменным.

Не следует понимать формулу для определения сопротивления в том смысле, что сопротивление данного проводника зависит оттока и напряжения. Известно, что оно зависит от длины, площади сечения и материала проводника. По внешнему виду формула для определения сопротивления напоминает формулу для расчета тока, но между ними имеется принципиальная разница. Ток в данном участке цепи действительно зависит от напряжения и сопротивления и изменяется при их изменении. А сопротивление данного участка цепи является величиной постоянной, не зависящей от изменения напряжения и тока, но равной отношению этих величин.

Когда один и тот же ток проходит в двух участках цепи, а напряжения, приложенные к ним, различны, то ясно, что участок, к которому приложено большее напряжение, имеет соответственно большее сопротивление. А если под действием одного и того же напряжения в двух разных участках цепи проходит различный ток, то меньший ток всегда будет на том участке, который имеет большее сопротивление. Все это вытекает из основной формулировки закона Ома для участка цепи, т. е. из того, что ток тем больше, чем больше напряжение и чем меньше сопротивление.

Расчет сопротивления с помощью закона Ома для участка цепи покажем на следующем примере. Пусть требуется найти сопротивление участка, через который при напряжении 40 В проходит ток 50 мА. Выразив ток в амперах, получим I = 0,05 А. Разделим 40 на 0,05 и найдем, что сопротивление составляет 800 Ом.

Закон Ома можно наглядно представить в виде так называемой вольт-амперной характеристики . Как известно, прямая пропорциональная зависимость между двумя величинами представляет собой прямую линию, проходящую через начало координат. Такую зависимость принято называть линейной .

Говорят: «не знаешь закон Ома – сиди дома». Так давайте же узнаем (вспомним), что это за закон, и смело пойдем гулять.

Основные понятия закона Ома

Как понять закон Ома? Нужно просто разобраться в том, что есть что в его определении. И начать следует с определения силы тока, напряжения и сопротивления.

Сила тока I

Пусть в каком-то проводнике течет ток. То есть, происходит направленное движение заряженных частиц – допустим, это электроны. Каждый электрон обладает элементарным электрическим зарядом (e= -1,60217662 × 10 -19 Кулона). В таком случае через некоторую поверхность за определенный промежуток времени пройдет конкретный электрический заряд, равный сумме всех зарядов протекших электронов.

Отношение заряда к времени и называется силой тока. Чем больший заряд проходит через проводник за определенное время, тем больше сила тока. Сила тока измеряется в Амперах .

Напряжение U, или разность потенциалов

Это как раз та штука, которая заставляет электроны двигаться. Электрический потенциал характеризует способность поля совершать работу по переносу заряда из одной точки в другую. Так, между двумя точками проводника существует разность потенциалов, и электрическое поле совершает работу по переносу заряда.

Физическая величина, равная работе эффективного электрического поля при переносе электрического заряда, и называется напряжением. Измеряется в Вольтах . Один Вольт – это напряжение, которое при перемещении заряда в 1 Кл совершает работу, равную 1 Джоуль .

Сопротивление R

Ток, как известно, течет в проводнике. Пусть это будет какой-нибудь провод. Двигаясь по проводу под действием поля, электроны сталкиваются с атомами провода, проводник греется, атомы в кристаллической решетке начинают колебаться, создавая электронам еще больше проблем для передвижения. Именно это явление и называется сопротивлением. Оно зависит от температуры, материала, сечения проводника и измеряется в Омах .


Формулировка и объяснение закона Ома

Закон немецкого учителя Георга Ома очень прост. Он гласит:

Сила тока на участке цепи прямо пропорционально напряжению и обратно пропорциональна сопротивлению.

Георг Ом вывел этот закон экспериментально (эмпирически) в 1826 году. Естественно, чем больше сопротивление участка цепи, тем меньше будет сила тока. Соответственно, чем больше напряжение, тем и ток будет больше.

Кстати! Для наших читателей сейчас действует скидка 10% на

Данная формулировка закона Ома – самая простая и подходит для участка цепи. Говоря “участок цепи” мы подразумеваем, что это однородный участок, на котором нет источников тока с ЭДС. Говоря проще, этот участок содержит какое-то сопротивление, но на нем нет батарейки, обеспечивающей сам ток.

Если рассматривать закон Ома для полной цепи, формулировка его будет немного иной.

Пусть у нас есть цепь, в ней есть источник тока, создающий напряжение, и какое-то сопротивление.

Закон запишется в следующем виде:

Объяснение закона Ома для полой цепи принципиально не отличается от объяснения для участка цепи. Как видим, сопротивление складывается из собственно сопротивления и внутреннего сопротивления источника тока, а вместо напряжения в формуле фигурирует электродвижущая сила источника.

Кстати, о том, что такое что такое ЭДС , читайте в нашей отдельной статье.

Как понять закон Ома?

Чтобы интуитивно понять закон Ома, обратимся к аналогии представления тока в виде жидкости. Именно так думал Георг Ом, когда проводил опыты, благодаря которым был открыт закон, названный его именем.

Представим, что ток – это не движение частиц-носителей заряда в проводнике, а движение потока воды в трубе. Сначала воду насосом поднимают на водокачку, а оттуда, под действием потенциальной энергии, она стремиться вниз и течет по трубе. Причем, чем выше насос закачает воду, тем быстрее она потечет в трубе.

Отсюда следует вывод, что скорость потока воды (сила тока в проводе) будет тем больше, чем больше потенциальная энергия воды (разность потенциалов)

Сила тока прямо пропорциональна напряжению.

Теперь обратимся к сопротивлению. Гидравлическое сопротивление – это сопротивление трубы, обусловленное ее диаметром и шероховатостью стенок. Логично предположить, что чем больше диаметр, тем меньше сопротивление трубы, и тем большее количество воды (больший ток) протечет через ее сечение.

Сила тока обратно пропорциональна сопротивлению.

Такую аналогию можно проводить лишь для принципиального понимания закона Ома, так как его первозданный вид – на самом деле довольно грубое приближение, которое, тем не менее, находит отличное применение на практике.

В действительности, сопротивление вещества обусловлено колебанием атомов кристаллической решетки, а ток – движением свободных носителей заряда. В металлах свободными носителями являются электроны, сорвавшиеся с атомных орбит.


В данной статье мы постарались дать простое объяснение закона Ома. Знание этих на первый взгляд простых вещей может сослужить Вам неплохую службу на экзамене. Конечно, мы привели его простейшую формулировку закона Ома и не будем сейчас лезть в дебри высшей физики, разбираясь с активным и реактивным сопротивлениями и прочими тонкостями.

Если у Вас возникнет такая необходимость, Вам с удовольствием помогут сотрудники нашего . А напоследок предлагаем Вам посмотреть интересное видео про закон Ома. Это действительно познавательно!

Закон Ома часто называют основным законом электричества. Открывший его в 1826 г. известный немецкий физик Георг Симон Ом установил зависимость между основными физическими величинами электрической цепи – сопротивлением, напряжением и силой тока.

Электрическая цепь

Чтобы лучше понять смысл закона Ома, нужно представлять, как устроена электрическая цепь.

Что же такое электрическая цепь? Это путь, который проходят электрически заряженные частицы (электроны) в электрической схеме.

Чтобы в электрической цепи существовал ток, необходимо наличие в ней устройства, которое создавало бы и поддерживало разность потенциалов на участках цепи за счёт сил неэлектрического происхождения. Такое устройство называется источником постоянного тока , а силы – сторонними силами .

Электрическую цепь, в которой находится источник тока, называют полной электрической цепью . Источник тока в такой цепи выполняет примерно такую же функцию, что и насос, перекачивающий жидкость в замкнутой гидравлической системе.

Простейшая замкнутая электрическая цепь состоит из одного источника и одного потребителя электрической энергии, соединённых между собой проводниками.

Параметры электрической цепи

Свой знаменитый закон Ом вывел экспериментальным путём.

Проведём несложный опыт.

Соберём электрическую цепь, в которой источником тока будет аккумулятор, а прибором для измерения тока – последовательно включенный в цепь амперметр. Нагрузкой служит спираль из проволоки. Напряжение будем измерять с помощью вольтметра, включенного параллельно спирали. Замкнём с помощью ключа электрическую цепь и запишем показания приборов.

Подключим к первому аккумулятору второй с точно таким же параметрами. Снова замкнём цепь. Приборы покажут, что и сила тока, и напряжение увеличились в 2 раза.

Если к 2 аккумуляторам добавить ещё один такой же, сила тока увеличится втрое, напряжение тоже утроится.

Вывод очевиден: сила тока в проводнике прямо пропорциональна напряжению, приложенному к концам проводника .

В нашем опыте величина сопротивления оставалась постоянной. Мы меняли лишь величину тока и напряжения на участке проводника. Оставим лишь один аккумулятор. Но в качестве нагрузки будем использовать спирали из разных материалов. Их сопротивления отличаются. Поочерёдно подключая их, также запишем показания приборов. Мы увидим, что здесь всё наоборот. Чем больше величина сопротивления, тем меньше сила тока. Сила тока в цепи обратно пропорциональна сопротивлению .

Итак, наш опыт позволил нам установить зависимость силы тока от величины напряжения и сопротивления.

Конечно, опыт Ома был другим. В те времена не существовало амперметров, и, чтобы измерить силу тока, Ом использовал крутильные весы Кулона. Источником тока служил элемент Вольта из цинка и меди, которые находились в растворе соляной кислоты. Медные проволоки помещались в чашки со ртутью. Туда же подводились концы проводов от источника тока. Проволоки были одинакового сечения, но разной длины. За счёт этого менялась величина сопротивления. Поочерёдно включая в цепь различные проволоки, наблюдали за углом поворота магнитной стрелки в крутильных весах. Собственно, измерялась не сама сила тока, а изменение магнитного действия тока за счёт включения в цепь проволок различного сопротивления. Ом называл это «потерей силы».

Но так или иначе эксперименты учёного позволили ему вывести свой знаменитый закон.

Георг Симон Ом

Закон Ома для полной цепи

Между тем, формула, выведенная самим Омом, выглядела так:

Это не что иное, как формула закона Ома для полной электрической цепи: « Сила тока в цепи пропорциональна действующей в цепи ЭДС и обратно пропорциональна сумме сопротивлений внешней цепи и внутреннего сопротивления источника ».

В опытах Ома величина Х показывала изменение величины тока. В современной формуле ей соответствует сила тока I , протекающего в цепи. Величина а характеризовала свойства источника напряжения, что соответствует современному обозначению электродвижущей силы (ЭДС) ε . Значение величины l зависело от длины проводников, соединявших элементы электрической цепи. Эта величина являлась аналогией сопротивления внешней электрической цепи R . Параметр b характеризовал свойства всей установки, на которой проводился опыт. В современной обозначении это r – внутреннее сопротивление источника тока.

Как выводится современная формула закона Ома для полной цепи?

ЭДС источника равна сумме падений напряжений на внешней цепи (U ) и на самом источнике (U 1 ).

ε = U + U 1 .

Из закона Ома I = U / R следует, что U = I · R , а U 1 = I · r .

Подставив эти выражения в предыдущее, получим:

ε = I · R + I · r = I · (R + r) , откуда

По закону Ома напряжение во внешней цепи равно произведению силы тока на сопротивление. U = I · R . Оно всегда меньше, чем ЭДС источника. Разница равна величине U 1 = I · r .

Что происходит при работе батарейки или аккумулятора? По мере того, как разряжается батарейка, растёт её внутренне сопротивление. Следовательно, увеличивается U 1 и уменьшается U .

Полный закон Ома превращается в закон Ома для участка цепи, если убрать из него параметры источника.

Короткое замыкание

А что произойдёт, если сопротивление внешней цепи вдруг станет равно нулю? В повседневной жизни мы можем наблюдать это, если, например, повреждается электрическая изоляция проводов, и они замыкаются между собой. Возникает явление, которое называется коротким замыканием . Ток, называемый током короткого замыкания , будет чрезвычайно большим. При этом выделится большое количество теплоты, которое может привести к пожару. Чтобы этого не случилось, в цепи ставят устройства, называемые предохранителями. Они устроены так, что способны разорвать электрическую цепь в момент короткого замыкания.

Закон Ома для переменного тока

В цепи переменного напряжения кроме обычного активного сопротивления встречается реактивное сопротивление (ёмкости, индуктивности).

Для таких цепей U = I · Z , где Z – полное сопротивление, включающее в себя активную и реактивную составляющие.

Но большим реактивным сопротивлением обладают мощные электрические машины и силовые установки. В бытовых приборах, окружающих нас, реактивная составляющая настолько мала, что её можно не учитывать, а для расчётов использовать простую форму записи закона Ома:

I = U / R

Мощность и закон Ома

Ом не только установил зависимость между напряжением, током и сопротивлением электрической цепи, но и вывел уравнение для определения мощности:

P = U · I = I 2 · R

Как видим, чем больше ток или напряжение, тем больше мощность . Так как проводник или резистор не является полезной нагрузкой, то мощность, которая приходится на него, считается мощностью потерь. Она идёт на нагревание проводника. И чем больше сопротивление такого проводника, тем больше теряется на нём мощности. Чтобы уменьшить потери от нагревания, в цепи используют проводники с меньшим сопротивлением. Так делают, например, в мощных звуковых установках.

Вместо эпилога

Небольшая подсказка для тех, кто путается и не может запомнить формулу закона Ома.

Разделим треугольник на 3 части. Причём, каким образом мы это сделаем, совершенно неважно. Впишем в каждую из них величины, входящие в закон Ома – так, как показано на рисунке.

Закроем величину, которую нужно найти. Если оставшиеся величины находятся на одном уровне, то их нужно перемножить. Если же они располагаются на разных уровнях, то величину, расположенную выше, необходимо разделить на нижнюю.

Закон Ома широко применяется на практике при проектировании электрических сетей в производстве и в быту.

Зависит величина воздействия, которое ток может оказывать на проводник, будь то тепловое, химическое или магнитное действие тока . То есть, регулируя силу тока, можно управлять его воздействием. Электрический ток , в свою очередь – это упорядоченное движение частиц под действием электрического поля .

Зависимость силы тока и напряжения

Очевидно, что чем сильнее поле действует на частицы, тем больше будет сила тока в цепи. Электрическое поле характеризуется величиной, называемой напряжением . Следовательно, мы приходит к выводу, что сила тока зависит от напряжения.

И действительно, опытным путем удалось установить, что сила тока связана с напряжением прямо пропорционально. В случаях, когда изменяли величину напряжения в цепи, не меняя всех остальных параметров, сила тока возрастала или уменьшалась во столько же раз, во сколько меняли напряжение.

Связь с сопротивлением

Однако любая цепь или участок цепи характеризуются еще одной немаловажной величиной, называемой сопротивлением электрическому току . Сопротивление связано с силой тока обратно пропорционально. Если на каком-либо участке цепи изменить величину сопротивления, не меняя напряжения на концах этого участка, сила тока также изменится. Причем если мы уменьшим величину сопротивления, то сила тока возрастет во столько же раз. И, наоборот, при увеличении сопротивления сила тока пропорционально уменьшается.

Формула закона Ома для участка цепи

Сопоставив две эти зависимости, можно прийти к такому же выводу, к которому пришел немецкий ученый Георг Ом в 1827 г. Он связал воедино три вышеуказанные физические величины и вывел закон, который назвали его именем. Закон Ома для участка цепи гласит:

Сила тока в участке цепи прямо пропорциональна напряжению на концах этого участка и обратно пропорциональна его сопротивлению.

где I – сила тока,
U – напряжение,
R – сопротивление.

Применение закона Ома

Закон Ома – один из основополагающих законов физики . Открытие его в свое время позволило сделать огромный скачок в науке. В настоящее время невозможно себе представить любой самый элементарный расчет основных электрических величин для любой цепи без использования закона Ома. Представление об этом законе – это не удел исключительно инженеров-электронщиков, а необходимая часть базовых знаний любого мало-мальски образованного человека. Недаром есть поговорка: «Не знаешь закон Ома – сиди дома».

U=IR и R=U/I

Правда, следует понимать, что в собранной цепи величина сопротивления некоторого участка цепи есть величина постоянная, поэтому при изменении силы тока будет изменяться только напряжение и наоборот. Для изменения сопротивления участка цепи следует собрать цепь заново. Расчет же требуемой величины сопротивления при проектировании и сборке цепи можно произвести по закону Ома, исходя из предполагаемых значений силы тока и напряжения, которые будут пропущены через данный участок цепи.

Закон

Ома – Как соотносятся напряжение, ток и сопротивление | Закон Ома

Первая и, возможно, самая важная взаимосвязь между током, напряжением и сопротивлением называется законом Ома, который был открыт Георгом Симоном Омом и опубликован в его статье 1827 года «Гальваническая цепь, исследованная математически».

Напряжение, ток и сопротивление

Электрическая цепь образуется, когда создается проводящий путь, позволяющий электрическому заряду непрерывно перемещаться. Это непрерывное движение электрического заряда через проводники цепи называется током , и его часто называют «потоком», как поток жидкости через полую трубу.

Сила, побуждающая носители заряда «течь» в цепи, называется напряжением . Напряжение – это особая мера потенциальной энергии, которая всегда относительна между двумя точками.

Когда мы говорим об определенном количестве напряжения, присутствующем в цепи, мы имеем в виду измерение того, сколько потенциальной энергии существует для перемещения носителей заряда из одной конкретной точки в этой цепи в другую конкретную точку. Без ссылки на две конкретные точки термин «напряжение» не имеет значения.

Ток имеет тенденцию проходить через проводники с некоторой степенью трения или сопротивления движению. Это противодействие движению правильнее называть сопротивлением . Сила тока в цепи зависит от величины напряжения и величины сопротивления в цепи, препятствующей прохождению тока.

Как и напряжение, сопротивление – это величина, относительная между двумя точками. По этой причине величины напряжения и сопротивления часто указываются как «между» или «поперек» двух точек в цепи.

Единицы измерения: вольт, ампер и ом

Чтобы иметь возможность делать осмысленные утверждения об этих величинах в цепях, мы должны уметь описывать их количества так же, как мы могли бы количественно определить массу, температуру, объем, длину или любой другой вид физической величины. Для массы мы можем использовать единицы «килограмм» или «грамм».

Для температуры мы можем использовать градусы Фаренгейта или градусы Цельсия. Вот стандартные единицы измерения электрического тока, напряжения и сопротивления:

«Символ», присвоенный каждой величине, представляет собой стандартную буквенную букву, используемую для представления этой величины в алгебраическом уравнении. Подобные стандартизированные буквы распространены в физических и технических дисциплинах и признаны во всем мире.

«Аббревиатура единицы» для каждой величины представляет собой алфавитный символ, используемый в качестве сокращенного обозначения для ее конкретной единицы измерения. И да, этот странно выглядящий символ «подкова» – это заглавная греческая буква Ω, просто символ иностранного алфавита (извинения перед читателями-греками).

Каждая единица измерения названа в честь известного экспериментатора в области электричества: amp в честь француза Андре М.Ампер, вольт после итальянского Алессандро Вольта и Ом после немца Георга Симона Ома.

Математический символ для каждой величины также имеет значение. «R» для сопротивления и «V» для напряжения говорят сами за себя, тогда как «I» для тока кажется немного странным. Считается, что буква «I» должна обозначать «интенсивность» (потока заряда), а другой символ напряжения, «E», означает «электродвижущую силу». Судя по исследованиям, которые мне удалось провести, кажется, что есть некоторые разногласия по поводу значения слова «Я».”

Символы «E» и «V» по большей части взаимозаменяемы, хотя в некоторых текстах зарезервировано «E» для обозначения напряжения на источнике (таком как батарея или генератор) и «V» для обозначения напряжения на любом другом элементе.

Все эти символы выражаются заглавными буквами, за исключением случаев, когда величина (особенно напряжение или ток) описывается в терминах короткого периода времени (называемого «мгновенным» значением). Например, напряжение батареи, которое стабильно в течение длительного периода времени, будет обозначаться заглавной буквой «E», в то время как пик напряжения при ударе молнии в тот самый момент, когда он попадает в линию электропередачи, скорее всего, будет обозначается строчной буквой «е» (или строчной буквой «v»), чтобы обозначить это значение как имеющееся в один момент времени.

То же самое соглашение о нижнем регистре справедливо и для тока, строчная буква «i» представляет ток в некоторый момент времени. Однако большинство измерений постоянного тока (DC), которые стабильны во времени, будут обозначены заглавными буквами.

Кулон и электрический заряд

Одна из основополагающих единиц электрического измерения, которую часто преподают в начале курсов электроники, но нечасто используют впоследствии, – это единица кулонов , которая представляет собой меру электрического заряда, пропорционального количеству электронов в несбалансированном состоянии.Один кулон заряда равен 6 250 000 000 000 000 000 электронов.

Символом количества электрического заряда является заглавная буква «Q», а единица измерения кулонов обозначается заглавной буквой «C». Бывает так, что единица измерения тока, ампер, равна 1 кулону заряда, проходящего через заданную точку в цепи за 1 секунду. В этих терминах ток – это скорость движения электрического заряда по проводнику.

Как указывалось ранее, напряжение является мерой потенциальной энергии на единицу заряда , доступной для стимулирования протекания тока из одной точки в другую. Прежде чем мы сможем точно определить, что такое «вольт», мы должны понять, как измерить эту величину, которую мы называем «потенциальной энергией». Общая метрическая единица для энергии любого вида – джоулей , равная количеству работы, совершаемой силой в 1 ньютон при движении на 1 метр (в том же направлении).

В британских подразделениях это чуть меньше 3/4 фунта силы, приложенной на расстоянии 1 фута. Проще говоря, требуется около 1 джоуля энергии, чтобы поднять гирю весом 3/4 фунта на 1 фут от земли или перетащить что-то на расстояние 1 фут, используя параллельную тяговую силу 3/4 фунта.В этих научных терминах 1 вольт равен 1 джоулю электрической потенциальной энергии на (деленный на) 1 кулон заряда. Таким образом, 9-вольтовая батарея выделяет 9 джоулей энергии на каждый кулон заряда, проходящего через цепь.

Эти единицы и символы электрических величин станут очень важны, когда мы начнем исследовать отношения между ними в цепях.

Уравнение закона Ома

Принципиальное открытие

Ома заключалось в том, что величина электрического тока, протекающего через металлический проводник в цепи, прямо пропорциональна напряжению, приложенному к нему при любой заданной температуре. Ом выразил свое открытие в виде простого уравнения, описывающего взаимосвязь напряжения, тока и сопротивления:

В этом алгебраическом выражении напряжение (E) равно току (I), умноженному на сопротивление (R). Используя методы алгебры, мы можем преобразовать это уравнение в два варианта, решая для I и R соответственно:

Анализ простых схем с помощью закона Ома

Давайте посмотрим, как эти уравнения могут работать, чтобы помочь нам анализировать простые схемы:

В приведенной выше схеме есть только один источник напряжения (батарея слева) и только один источник сопротивления току (лампа справа).Это позволяет очень легко применять закон Ома. Если мы знаем значения любых двух из трех величин (напряжения, тока и сопротивления) в этой цепи, мы можем использовать закон Ома для определения третьей.

В этом первом примере мы рассчитаем величину тока (I) в цепи, учитывая значения напряжения (E) и сопротивления (R):

Какой ток (I) в этой цепи?

В этом втором примере мы рассчитаем величину сопротивления (R) в цепи, учитывая значения напряжения (E) и тока (I):

Какое сопротивление (R) дает лампа?

В последнем примере мы рассчитаем величину напряжения, подаваемого батареей, с учетом значений тока (I) и сопротивления (R):

Какое напряжение обеспечивает аккумулятор?

Техника треугольника закона Ома

Закон Ома – очень простой и полезный инструмент для анализа электрических цепей. Он так часто используется при изучении электричества и электроники, что серьезный студент должен запомнить его. Для тех, кто еще не знаком с алгеброй, есть уловка, чтобы запомнить, как найти любое количество, учитывая два других.

Сначала расположите буквы E, I и R в виде треугольника следующим образом:

Если вы знаете E и I и хотите определить R, просто удалите R с картинки и посмотрите, что осталось:

Если вы знаете E и R и хотите определить I, удалите I и посмотрите, что осталось:

Наконец, если вы знаете I и R и хотите определить E, удалите E и посмотрите, что осталось:

В конце концов, вам придется быть знакомым с алгеброй, чтобы серьезно изучать электричество и электронику, но этот совет может облегчить запоминание ваших первых вычислений.Если вы хорошо разбираетесь в алгебре, все, что вам нужно сделать, это зафиксировать E = IR в памяти и вывести из нее две другие формулы, когда они вам понадобятся!

ОБЗОР:

  • Напряжение измеряется в вольт , обозначается буквами «E» или «V».
  • Ток измеряется в ампер , обозначается буквой «I».
  • Сопротивление измеряется в Ом. обозначается буквой «R».
  • Закон Ома: E = IR; I = E / R; R = E / I

СВЯЗАННЫЕ РАБОЧИЕ ЛИСТЫ:

Попробуйте наш калькулятор закона Ома в разделе «Инструменты».

Формула закона

Ома и практика расчета цепей – видео и стенограмма урока

Пример цепи

Давайте начнем с этой подсказки – определим следующее:

  1. Ток через резистор R
  2. Номинал резистора R
  3. Напряжение на резисторе 10 Ом
  4. Токи через обе ветви
Принципиальная электрическая схема

Вот наше решение.Давайте сначала признаем, что это обширный список вещей, которые нужно сделать. Вся проблема кажется большим запутанным узлом. Развязывая запутанный узел, вы выбираете начальную точку и развязываете по одной петле за раз. Вот что мы сделаем с этой проблемой, и первым шагом будет начать с числа 1 и определить ток через резистор R.

Из принципиальной схемы видно, что ток в 7 ампер проходит через неизвестный резистор. . Теперь мы можем использовать уравнение мощности вместе с законом Ома для определения сопротивления неизвестного резистора R.

P = IV

  • P – мощность в ваттах.
  • I – ток в амперах (или амперах).
  • В – напряжение в вольтах (или В).

Закон Ома равен В = IR .

  • В – вольт.
  • I – текущий.
  • R – сопротивление в Ом (или Ω)

Мы знаем ток и мощность через резистор R.Давайте подключим IR к V в уравнении мощности.

Теперь мы можем решить это уравнение для R и получить значение резистора R.

Ответ на номер 2

Большая часть анализа схем требует правил Кирхгофа. Мы начнем с правила Кирхгофа , которое основано на сохранении электрического заряда и гласит, что ток, входящий в соединение, должен быть равен току, выходящему из соединения.Схема на вашем экране (ниже) показывает эти перекрестки.

Диаграмма 1

Напишем уравнения для токов на каждом переходе.

Сначала перекресток 1. Как видите, i 1 + i 2 = 7.

я 1 + я 2 = 7

А теперь перекресток 2. Как видите, i 3 + i 4 = 7.

я 3 + я 4 = 7

Глядя на два уравнения, мы видим, что есть четыре неизвестных тока. Это означает, что нам нужно решить четыре уравнения для всех неизвестных переменных. Мы воспользуемся правилом цикла Кирхгофа, чтобы написать еще два уравнения. Правило петли Кирхгофа гласит, что сложение всех напряжений в замкнутом контуре должно равняться нулю. Это как подняться по лестнице, а затем вернуться туда, откуда вы начали.Ваше изменение высоты равно нулю. Электрический потенциал работает точно так же. Начало в одной точке цепи и возвращение в эту же точку не дает чистого изменения потенциала. ΔV = 0.

Две необходимые нам петли показаны на диаграмме на вашем экране ниже:

Диаграмма 2

Важно знать, что есть и другие петли, которые можно использовать. Однако для любого замкнутого контура, если ток идет в том же направлении, что и контур, напряжение на резисторе отрицательное.Если ток идет в направлении, противоположном петле, напряжение на резисторе положительное. Это похоже на ходьбу в гору или спуск. Если петля движется по течению, это можно сравнить с ходьбой под уклон. Уменьшение напряжения происходит так же, как и уменьшение высоты. Если петля движется против течения, это можно сравнить с ходьбой в гору. Напряжение увеличивается точно так же, как увеличивается высота над уровнем моря.

Поскольку в контурах есть только напряжения, нам понадобится закон Ома, IR , чтобы выразить напряжения на резисторах.Давайте составим уравнения для двух контуров на диаграмме.

Начнем с цикла 1. Начиная с левого нижнего угла по часовой стрелке, мы получаем -30 i 1 + 15 i 2 = 0.

Теперь перейдем к циклу 2. Начиная с нижнего левого угла под батареей и двигаясь по часовой стрелке, следуя зеленой стрелке, мы получаем 100 + (-30 i 1) + (-8,4) + (-10 i 4) = 0.

Обратите внимание, что мы переходим от отрицательной клеммы к положительной клемме батареи, что означает положительное изменение потенциала.

Давайте систематизируем все уравнения в виде диаграммы на вашем экране (ниже), чтобы мы могли понять, с чего начать! Это похоже на головоломку. Поиск угловых частей головоломки может помочь запустить процесс, точно так же, как составление уравнений может помочь нам понять, с чего начать.

График 1

Обратите внимание, что все переменные, которые подсказка хочет, чтобы мы определяли, находятся на этой диаграмме. Давайте начнем процесс получения этих значений, взяв уравнения Junction 1 и Loop 1 и решив для i 1 и i 2.Мы начинаем с умножения Junction 1 на 30, так что i 1 сократится, когда мы сложим эти два уравнения.

Добавление нового уравнения Junction 1 к уравнению Loop 1 дает нам i 2, которое является нашим решением 210 + -30 i 1 + 15 i 2 = 0, поэтому 45 i 2 = 210.

i 2 ≈ 4,67 А, то есть ток через резистор 15 Ом.Подставляя 4,67 в i 2 в уравнении соединения 1, мы получаем i 1 ≈ 2,33 A.

Мы можем подключить i 1 к уравнению 2 цикла, чтобы получить i 4.

Уравнение соединения 2 вместе с i 4 дает нам i 3, что составляет 7 – 2,17 = 4,83 А.

Чтобы получить напряжение на резисторе 10 Ом, мы используем закон Ома. Напряжение составляет (2,17 А) (10 Ом), что составляет 21,7 вольт.

Резюме урока

Давайте рассмотрим то, что мы узнали.Анализ комбинированных схем требует использования закона Ома , В, = IR и обоих правил Кирхгофа.

  • Правило Кирхгофа гласит, что ток в переходе должен быть равен току на выходе.
  • Правило петли Кирхгофа гласит, что сумма всех напряжений в замкнутом контуре равна нулю. Таблица напряжений на экране (ниже) показывает, как определить, являются ли напряжения на резисторах отрицательными или положительными.
Диаграмма напряжения

Для аккумуляторов напряжения указаны как отрицательные при перемещении от положительной клеммы к отрицательной и положительные при перемещении от отрицательной клеммы к положительной.

Закон

Ома и закон Ватта – Basic HVAC

Нажмите кнопку воспроизведения на следующем аудиоплеере, чтобы слушать, как вы читаете этот раздел.

В этом разделе дается краткое описание двух наиболее фундаментальных электрических соотношений: закона Ома , который описывает ток в электрических цепях, и закона Ватта , который описывает, как рассеивается мощность.

Объединив элементы напряжения , тока и сопротивления , Джордж Ом разработал следующую формулу:

[латекс] \ text {I} = \ dfrac {\ text {E}} {\ text {R}} [/ latex]

Где:

  • E = Напряжение в вольтах
  • I = ток в амперах
  • R = Сопротивление в Ом

Это называется законом Ома.

Допустим, у нас есть цепь с потенциалом 1 вольт, током 1 ампер и сопротивлением 1 Ом.Используя закон Ома, мы можем сказать:

[латекс] 1 \ text {A} = \ dfrac {1 \ text {V}} {1 \ text {ohm}} [/ латекс]

Допустим, это резервуар с широким шлангом. Количество воды в баке определяется как 1 вольт, а «узость» (сопротивление потоку) шланга определяется как 1 Ом. Используя закон Ома, это дает нам ток (ток) в 1 ампер.

Используя эту аналогию, давайте теперь рассмотрим резервуар с узким шлангом. Поскольку шланг более узкий, его сопротивление потоку выше. Определим это сопротивление как 2 Ом.Количество воды в резервуаре такое же, как и в другом резервуаре, поэтому, используя закон Ома, наше уравнение для резервуара с узким шлангом будет:

[латекс]? = \ Dfrac {1 \ text {V}} {2 \ text {ohms}} [/ латекс]

а какой ток? Поскольку сопротивление больше и напряжение такое же, это дает нам значение тока 0,5 А:

[латекс] 0,5 \ text {A} = \ dfrac {1 \ text {V}} {2 \ text {ohms}} [/ латекс]

Комбинируя элементы напряжение , ток и мощность , названный в честь Джеймса Ватта, закон Ватта определяется как следующая формула:

[латекс] \ text {P} = \ text {E} * \ text {I} [/ latex]

Где:

  • P = Мощность в ваттах
  • E = Напряжение в вольтах
  • I = ток в амперах

Электрическая мощность – это скорость передачи энергии. Он измеряется в джоулях в секунду (Дж / с). Один джоуль работы, выполняемой каждую секунду, означает, что мощность рассеивается со скоростью, равной одному ватт (Вт) .

Учитывая несколько известных нам основных терминов, связанных с электричеством, как мы можем рассчитать мощность в цепи?

Итак, у нас есть стандартное измерение электродвижущей силы, также известное как напряжение (E) .

Ток, еще один из наших любимых электрических терминов, измеряет поток заряда во времени в единицах ампер (А) , что равно 1 кулону в секунду (Кл / с).Соедините их вместе, и что мы получим? Сила!

Чтобы рассчитать мощность любого конкретного компонента в цепи, умножьте падение напряжения на нем на ток, протекающий через него.

Например, если ток течет со скоростью 10 ампер, а доступное напряжение составляет 10 вольт, тогда схема рассеивает мощность со скоростью 100 Вт.

[латекс] 100 \ text {W} = 10 \ text {V} * 10 \ text {A} [/ latex]

Текстовые ссылки

Закон

Ома | Основы резистора

Что такое закон Ома?

Закон

Ома гласит, что электрический ток через проводник пропорционален разности потенциалов на нем. Кроме того, электрическое сопротивление проводника постоянно. Это приводит к математическому уравнению:

$$ R = \ frac {V} {I} $$

, где R, – сопротивление в омах (Ом), В, – напряжение в вольтах (В), а I – ток в амперах (А). Для иллюстрации: резистор сопротивлением 1 Ом, на который действует ток 1 А, имеет разность напряжений на своих выводах 1 В. Уравнение названо в честь Георга Ома. В 1827 году он опубликовал свои выводы, которые легли в основу формулы, которая используется сегодня.Он провел большую серию экспериментов, которые показали связь между приложенным напряжением и током, протекающим через проводник. Следовательно, закон эмпирический. Хотя закон Ома является одной из основ электротехники, на момент публикации он был встречен с критикой. Ом принят в качестве официальной единицы измерения электрического сопротивления в системе СИ. Густав Кирхгоф (известный из законов схем Кирхгофа) сделал обобщение, которое чаще используется в физике:

$$ \ sigma = \ frac {J} {E} $$

, где σ – параметр проводимости (зависит от материала), Дж, – плотность тока, а E – электрическое поле.

Закон Ома и резисторы

Резисторы – это пассивные элементы, которые создают сопротивление прохождению электрического тока в цепи. Резистор, работающий по закону Ома, называется омическим резистором. Когда ток проходит через омический резистор, падение напряжения на выводах пропорционально величине сопротивления. Формула Ома действительна также для цепей с переменным напряжением или током, поэтому ее можно использовать и для цепей переменного тока.Для конденсаторов и катушек индуктивности нельзя использовать закон Ома, поскольку их ВАХ по своей природе нелинейны (не омичны).

Формула

Ома действительна для цепей с несколькими резисторами, которые можно соединить последовательно, параллельно или и то, и другое. Группы резисторов, включенных последовательно или параллельно, можно упростить с помощью эквивалентного сопротивления. В статьях «Последовательные резисторы» и «Параллельные резисторы» более подробно описан этот процесс.

Георг Симон Ом (1789-1854)

Георг Симон Ом

В 1827 году немецкий физик Георг Симон Ом опубликовал свою полную теорию электричества под названием Гальваническая цепь, исследованная математически . Он обнаружил, что падение напряжения в части цепи является произведением проходящего через нее тока и сопротивления этой части. Это легло в основу того закона, которым мы пользуемся сегодня. Закон – одно из основополагающих соотношений для резисторов.

Его коллеги не оценили его выводы, и закон был нелегко принят. Ом был учителем в гимназии в Кельне в то время, и он решил уйти в отставку. Ом стал профессором экспериментальной физики в Мюнхенском университете. Позже он был наконец признан за свою работу и получил медаль Копли в 1841 году от Королевского общества.

Уравнения закона Ома

Формула

Ома может использоваться, когда известны две из трех переменных. Связь между сопротивлением, током и напряжением можно записать по-разному. Чтобы запомнить это, может оказаться полезным калькулятор треугольника Ом, показанный на рисунке. Два примера ниже покажут использование калькулятора треугольника и закона Ома.

$$ R = \ frac {V} {I} $$

или

$$ V = I · R $$

или

$$ I = \ frac {V} {R} $$

Примеры использования закона Ома

Рассмотрим резистор 1 Ом в цепи с падением напряжения на его выводах от 100 В до 10 В. Какой ток через резистор?

Треугольник напоминает нам, что: $$ I = \ frac {V} {R} = \ frac {100 – 10} {1} = 90 A $$

Рассмотрим резистор 10 Ом в цепи, подверженной току 2 А и напряжению 120 В. Какое падение напряжения на резисторе?

Использование треугольника показывает нам, что:

$$ V = I · R = 2 · 10 = 20 В $$

Падение напряжения на резисторах 20 В; следовательно, напряжение на оконечном выводе составляет 120-20 = 100 В.2} {R} $$

Идеальные резисторы рассеивают всю энергию и не накапливают электрическую или магнитную энергию. У каждого резистора есть предел мощности, которая может рассеиваться без повреждения. Это называется номинальной мощностью. Окружающие условия могут снизить это значение. Например, корпус вокруг резистора или более высокая температура окружающей среды уменьшат количество энергии, которое резистор может рассеять. 2 · R = (0.2 · 50 = 0,5 Вт $$

Минимальная номинальная мощность должна быть не менее 0,5 Вт, но рекомендуется выбирать резистор с номиналом значительно выше этого значения для дополнительной надежности и срока службы.

Какой ток в цепи?

Это основной пример закона Ома. Напряжение и сопротивление известны, поэтому мы можем рассчитать ток по формуле:

$$ I = \ frac {V} {R} = \ frac {6} {1.2} = 5 A $$

Электронагреватель (резистор) с потреблением 1 кВт включен в цепь с током 8 А. Какое падение напряжения на ТЭНе?

Напряжение можно рассчитать исходя из мощности и тока по формуле:

$$ V = \ frac {P} {I} = \ frac {1000} {8} = 125 В $$

Цветовой код резистора

Значение сопротивления в омах часто обозначается цветовым кодом на резисторе. Комбинация цветов указывает значение, а также допуск резистора. Для получения дополнительной информации по этой теме см. Цветовую кодировку резистора.

ЗАКОН О ТОКАХ И ОМС

ЗАКОН О ТОКАХ И ОМС
Рисунок 28.1. Электрическое поле в проводе. Когда провод подключен к клеммам аккумулятора, электрическое поле создается внутри провода (см. рисунок 28.1). Свобода электроны в проводе будут двигаться в направлении, противоположном направлению поля линий.Электрический заряд будет пытаться перераспределиться таким образом, чтобы чистое электрическое поле в проводе равно нулю. Однако положительный клемма батареи действует как сток для электронов, а отрицательная клемма действует как источник электронов, и непрерывный поток электронов будет созданный. Этот непрерывный поток электронов называется электрическим током. текущий . Символ тока – I, а его единица СИ – Ампер (А). Сила тока определена как

. (28.1)

где dq – количество заряда, которое проходит через некоторую заданную точку на проводе. в течение периода времени dt. Ток 1 А равен 1 Кл / с. В плотность тока Дж определяется как

(28,2)

где I – ток, протекающий по проводнику, а A – площадь поперечного сечения проводника. Хотя электроны чувствуют электрическое поле внутри проводника, они не будут ускоряться.Электроны будет испытывать значительное трение в результате столкновений с положительные ионы в проводнике. В среднем электроны будут двигаться с постоянная скорость от отрицательной клеммы аккумулятора к положительной Терминал. Их средняя скорость, также называемая дрейфовой скоростью v d , пропорционально электрическому полю E

(28,3)

При заданной плотности электронов в проводнике увеличение дрейфа скорость каждого из электронов увеличит количество проходящих электронов заданной точкой на проводнике в единицу времени.Это показано в Рисунок 28.2. За промежуток времени dt электроны пройдут в среднем около расстояние, равное dx, где

(28. 4)

Рисунок 28.2. Движение среднего электрона в проводнике. Все электроны на расстоянии dx от точки P будут поэтому пройти эту точку в течение временного интервала dt. Предположим, что плотность электронов в проводнике n электронов / м 3 . Номер электронов dN, которые пройдут через P за промежуток времени dt, тогда будет равно

(28.5)

Поскольку каждый электрон несет заряд e, полный заряд dQ, который пройдет точка P на временном интервале dt равна

(28,6)

Следовательно, ток через проводник равен

. (28,7)

Уравнение (28.7) показывает, что ток в проводнике пропорционален силе тока. площадь поперечного сечения проводника и пропорциональна скорости дрейфа. Поскольку скорость дрейфа пропорциональна электрическому полю E, следующее соотношение выполняется для тока в проводнике:

(28.8)

Электрическое поле в проводнике определяется его длиной L и величиной разность потенциалов [Дельта] V между двумя его концами (E = [Дельта] V / L). Таким образом, уравнение (28.8) можно переписать как

(28,9)

Уравнение (28.9) можно переписать как

(28.10)

Константа пропорциональности [rho] называется удельным сопротивлением материал. Удельное сопротивление [rho] зависит от характеристик проводник ([rho] маленький для хорошего проводника, и [rho] очень большой для хорошего проводника изолятор).Сопротивление R проводника определяется как

. (28.11)

Единицей измерения сопротивления в системе СИ является ом ([Омега]). Используя сопротивление R, мы можем перепишите уравнение (28.10)

(28.12)

Уравнение (28.12) называется Закон Ома . Уравнение (28.12) показывает что ток через проводник пропорционален потенциалу разница между концами проводника и обратно пропорциональна его сопротивление.Уравнение (28.12) также показывает, что 1 [Омега] равно 1 В / А.

Пример: Задача 28.5

Алюминиевый провод имеет сопротивление 0,10 Ом. Если вы нарисуете это провод через матрицу, сделав ее тоньше и вдвое длиннее, что будет ее новым сопротивление ?

Начальное сопротивление R и алюминиевого провода длиной L и площадь поперечного сечения A равна

(28,13)

Начальный объем провода – л. А. После прохождения провода через матрицу ее длина изменилась на L ‘, а площадь ее поперечного сечения равно A ‘. Таким образом, его конечный объем равен L ‘A’. Поскольку плотность алюминий не меняется, объем провода не меняется, и поэтому начальный и конечный размеры провода связаны:

(28,14)

или

(28.15)

Проблема заключается в том, что длина провода увеличена вдвое (L ‘= 2 L).В поэтому конечная площадь поперечного сечения A ‘связана с начальным площадь поперечного сечения A следующим образом:

(28,16)

Конечное сопротивление провода R f равно

. (28,17)

Сопротивление провода увеличилось в четыре раза и теперь составляет 0,40. [Омега].

Удельное сопротивление [rho] выражается в единицах ом-метр ([Omega] . м). В удельное сопротивление большинства проводников находится между 10 -8 [Омега] . м и 10 -7 [Omega] . г. Удельное сопротивление проводник зависит не только от типа материала, но и от его температура. Удельное сопротивление изолятора варьируется в пределах 10 11 [Omega] . м и 10 17 [Omega] . м. У всех материалов удельное сопротивление уменьшается с понижением температуры.В у некоторых материалов, таких как свинец, цинк, олово и ниобий, удельное сопротивление исчезает когда температура приближается к абсолютному нулю. При таких низких температурах эти материалы обладают сверхпроводимостью .

Пример: Задача 28.17

Кондиционер в доме потребляет ток 12 А. Предположим, что пара проводов, соединяющих кондиционер с блоком предохранителей, №10 медные провода диаметром 0. 259 см и длиной по 25 м.

а) Какое падение потенциала на каждом проводе? Предположим, что напряжение доставлено на дом ровно 110 В на блок предохранителей. Какое напряжение поставили кондиционер?

б) В некоторых старых домах используется медный провод № 12 диаметром 0,205. см. Повторите расчет части (а) для этого провода.

Рисунок 28.3. Электросхема кондиционера в проблеме 28.17.

а) Удельное сопротивление меди равно 1.7 x 10 -8 [Omega] . кв.м. (см. Таблицу 28.1). Сопротивление R Cu каждого медного провода равно на номер

(28.18)

где L – длина проволоки, а d – ее диаметр. Ток I есть протекает по проводам и I = 12 A. Падение напряжения [Delta] V на каждом провод равен

(28,19)

На рисунке 28.3 схематично показана электрическая схема кондиционера. схема.Напряжение на блоке кондиционера 110-2 . [Дельта] V, где [Дельта] V дается уравнением (28.19). Длина длина каждого медного кабеля составляет 25 м, а его диаметр равен 0,259 см. Напряжение падение на каждом проводе, таким образом, равно

(28.20)

Таким образом, напряжение на блоке переменного тока равно 108,1 В.

б) Проволока № 12 имеет диаметр 0,205 см. Падение напряжения на этот провод равен

(28.21)

а напряжение на блоке переменного тока равно 106,9 В.

Пример: Задача 28.12

ЛЭП высокого напряжения имеет алюминиевый кабель диаметром 3,0 мм. см, протяженностью 200 км. Какое сопротивление у этого кабеля?

Удельное сопротивление алюминия составляет 2,8 x 10 -8 Ом. длина кабель 200 км или 2 х 10 5 м. Диаметр кабеля 3 см. и его площадь поперечного сечения равна [пи] (d / 2) 2 или 7.1 х 10 -4 м 2 . Подставляя эти значения в уравнение (28.11), получаем сопротивление кабеля можно определить

(28.22)

Устройство, специально разработанное для обеспечения высокого сопротивления, называется резистор. Обозначение резистора на принципиальной схеме – зигзагообразная линия (см. Рисунок 28.4).

Рисунок 28.4. Символ резистора. На рисунке 28.5 показаны два резистора с сопротивлением R 1 и R 2 соединены последовательно.Предположим, что ток, протекающий через цепь равна I. Падение напряжения [Дельта] В 1 на резисторе R 1 равно

(28,23)

и падение напряжения [Delta] V 2 на резисторе R 2 составляет равно

(28,24)

Разность потенциалов [Delta] V в последовательной цепи равна

. (28,25)

Уравнение (28.25) показывает, что два последовательно включенных резистора действуют как один резистор с сопротивлением, равным сумме сопротивлений резистора 1 и сопротивление резистора 2

(28.26)

Рисунок 28.5. Два резистора соединены последовательно. На рис. 28.6 показаны два резистора, включенных параллельно. В В этой схеме ток через каждый резистор будет разным, но падение напряжения [Delta] V на каждом резисторе будет одинаковым. Используя закон Ома ток I 1 , протекающий через резистор R 1 , может быть вычислено

(28.27)

а ток I 2 , протекающий через резистор R 2 , равен на номер

(28.28)

Полный ток, протекающий по цепи, равен сумме токи через каждый резистор

(28.29)

Таким образом, схема резисторов, показанная на рисунке 28.6, эквивалентна одиночной резистор R, где R можно получить из следующего соотношения

(28.30)

Уравнение (28.30) показывает, что сопротивление параллельной комбинации резисторов всегда меньше сопротивления каждого в отдельности резисторы.

Рисунок 28.6. Два резистора подключены параллельно.

Пример: задача 28.41

Сверхпроводящие кабели промышленного производства состоят из нитей сверхпроводящего провода, заключенного в матрицу из меди. Пока нити сверхпроводящие, в них течет весь ток, и нет тока течет в медь.Но если сверхпроводимость внезапно выйдет из строя из-за повышение температуры, ток может пролиться на медь; это предотвращает повреждение нитей сверхпроводника. Рассчитайте сопротивление на метр длины медной матрицы. Медная матрица имеет диаметр 0,7 мм, а каждая из 2100 нитей имеет диаметр 0,01 мм.

Считайте 1 метр кабеля. Площадь поперечного сечения каждой нити накала [пи] . (д / 2) 2 = 7.9 x 10 -11 м 2 . В площадь поперечного сечения 2100 нитей равна 1,65 x 10 -7 м 2 . Диаметр медной матрицы равен 0,7 мм, а ее диаметр равен 0,7 мм. площадь поперечного сечения равна 1,54 х 10 -6 м 2 . В площадь самой меди, таким образом, равна 1,37 x 10 -6 м 2 . Сопротивление медной матрицы на единицу длины равно на номер

(28.31)

Предположим, что удельное сопротивление нити при комнатной температуре такое же, как у удельное сопротивление меди. Сопротивление каждой сверхпроводящей нити составляет равно

(28.32)

Провод можно рассматривать как параллельную цепь из одного резистора, представляющего сопротивление медной матрицы и 2100 резисторов, представляющих 2100 жилы сверхпроводящего провода. Доля тока, протекающего через медная матрица определяется легко.Предположим, что потенциал разность по проводнику равна [Delta] V. Электрический ток I Cu , протекающая через медную матрицу, равна

(28,33)

Ток I fil , протекающий через 2100 нитей, равен

. (28,34)

Доля F полного тока, протекающего через медную матрицу, равна на номер

(28.35)

Необходимо рассмотреть два особых случая.

1. Температура ниже критической. На уровне или ниже этого температура сопротивление нитей исчезает (R fil = 0 [Омега]). Уравнение (28.35) показывает, что в этом случае ток не будет течь. через медную матрицу.

2. Если температура провода выше критической температуры, текущий поток резко изменится. В этом случае доля ток, протекающий через медь, равен

(28.36)

Медная матрица будет пропускать 90% общего тока.

Пример: Задача 28.42

Какое сопротивление у комбинации из четырех резисторов, показанной на Рисунок 28.7. Каждый из резисторов имеет номинал R.

Рисунок 28.7. Проблема 28.42.

Чтобы найти сопротивление цепи, показанной на рисунке 28.7, мы начнем расчет чистого сопротивления R 34 параллельной цепи резисторы R 3 и R 4 :

(28.37)

или

(28,38)

Таким образом, схема, показанная на рисунке 28.7, эквивалентна показанной схеме. на рисунке 28.8. Резисторы R 2 и R 34 образуют серию сети и может быть заменен одиночным резистором с сопротивлением R 234 где

(28. 39)

Рисунок 28.8. Проблема 28.42. Рисунок 28.9. Проблема 28.42. Схема показана на рисунке 28.8 теперь можно заменить на Эквивалентная схема показана на рисунке 28.9. Сопротивление R до г. эта схема может быть получена из следующего соотношения

(28,40)

или

(28,41)

В рассматриваемом частном случае R 1 = R 2 = R 3 = R 4 = R. Таким образом,

(28,42)

(28.43)

(28,44)

Для R = 3 [Омега] полное сопротивление равно 1,8 [Омега].


Отправляйте комментарии, вопросы и / или предложения по электронной почте на адрес [email protected] и / или посетите домашнюю страницу Фрэнка Вольфса.

Что такое закон Ома? (Простое объяснение)

Что такое закон Ома?

Закон Ома гласит, что электрический ток, протекающий через любой проводник, прямо пропорционален разности потенциалов (напряжению) между его концами, при условии, что физические условия проводника не меняются.

Другими словами, отношение разности потенциалов между любыми двумя точками проводника к току, протекающему между ними, является постоянным при условии, что физические условия (например, температура и т. Д.) Не изменяются.

Математически закон Ома можно выразить как (),

  • I – ток через проводник в амперах (A),
  • V – напряжение или разность потенциалов, измеренная на проводнике в вольтах (В).
  • Закон Ома применим как к постоянному, так и к переменному току.

    Взаимосвязь между разностью потенциалов или напряжением (V), током (I) и сопротивлением (R) в электрической цепи впервые была обнаружена немецким физиком Джорджем Саймоном Омом.

    Единица сопротивления Ом () названа в честь Джорджа Саймона Ома.

    Как работает закон Ома?

    Согласно определению закона Ома, ток, протекающий через проводник или резистор между двумя точками, прямо пропорционален разности напряжения (или разности потенциалов) на проводнике или резисторе.

    Но… это может быть немного сложно понять.

    Итак, давайте лучше разберемся с законом Ома, используя некоторые аналогии.

    Аналогия 1

    Рассмотрим резервуар для воды, расположенный на определенной высоте над землей. Внизу резервуара для воды находится шланг, как показано на изображении ниже.

    Аналогия 1
    • Давление воды в паскалях на конце шланга аналогично напряжению или разности потенциалов в электрической цепи.
    • Расход воды в литрах в секунду аналогичен электрическому току в кулонах в секунду в электрической цепи.
    • Ограничители потока воды, такие как отверстия в трубах между двумя точками, аналогичны резисторам в электрической цепи.

    Таким образом, расход воды через ограничитель отверстия пропорционален разнице давления воды на ограничителе.

    Точно так же в электрической цепи ток, протекающий через проводник или резистор между двумя точками, прямо пропорционален разности напряжения или разности потенциалов на проводнике или резисторе.

    Мы также можем сказать, что сопротивление потоку воды зависит от длины трубы, материала трубы и высоты резервуара, установленного над землей.

    Ом работают в электрической цепи аналогичным образом, поскольку электрическое сопротивление, оказываемое протеканию тока, зависит от длины проводника и материала используемого проводника.

    Аналогия 2

    Простая аналогия между гидравлическим водяным контуром и электрической цепью для описания того, как работает закон Ома, показана на изображении ниже.

    Аналогия 2

    Как показано, если давление воды постоянно и ограничение увеличивается (что затрудняет прохождение воды), то скорость потока воды уменьшается.

    Точно так же в электрической цепи, если напряжение или разность потенциалов постоянны и сопротивление увеличивается (что затрудняет прохождение тока), то скорость протекания электрического заряда, т.е. ток уменьшается.

    Теперь, если ограничение потока воды постоянно и давление насоса увеличивается, скорость потока воды увеличивается.

    Точно так же в электрической цепи, если сопротивление постоянно и разность потенциалов или напряжение увеличивается, тогда скорость потока электрического заряда, то есть ток, увеличивается.

    Формула закона Ома

    Связь между напряжением или разностью потенциалов, током и сопротивлением можно записать тремя разными способами.

    Если нам известны два любых значения, мы можем вычислить третье неизвестное значение, используя соотношение по закону Ома. Таким образом, закон Ома очень полезен в электронике, электрических формулах и расчетах.

    Когда известный электрический ток протекает через известное сопротивление, падение напряжения на сопротивлении может быть вычислено по соотношению

    Когда известное напряжение приложено к известному сопротивлению, ток, текущий через сопротивление, может быть вычислен по соотношению

    Когда к неизвестному сопротивлению прикладывается известное напряжение и ток, протекающий через сопротивление, также известен, значение неизвестного сопротивления можно рассчитать по соотношению

    Формула закона Ома для мощности

    Передаваемая мощность равна произведение напряжения питания и электрического тока.

    (1)

    Теперь, подставив уравнение (1), мы получим,

    (2)

    Эта формула известна как формула омических потерь или формула резистивного нагрева.

    Теперь, подставив в уравнение (1), мы получим,

    (3)

    Из приведенного выше соотношения мы можем определить рассеиваемую мощность в сопротивлении, если известны либо напряжение и сопротивление, либо ток и сопротивление.

    Мы также можем определить неизвестное значение сопротивления, используя указанное выше соотношение, если известно напряжение или ток.

    Если известны любые две переменные мощности, напряжения, тока и сопротивления, то, используя закон Ома, мы можем определить две другие переменные.

    Ограничения закона Ома

    Некоторые ограничения закона Ома обсуждаются ниже.

    • Закон Ома распространяется не на все неметаллические проводники. Например, для карбида кремния соотношение определяется как где K и m – константы, а m <1.
    • Закон Ома не применим к следующим нелинейным элементам.
    1. Сопротивление
    2. Емкость
    3. Полупроводники
    4. Вакуумные трубки
    5. Электролиты
    6. Угольные резисторы
    7. Дуговые лампы
    8. Стабилитрон

    является нелинейным, т. е. ток не точно пропорционален приложенному напряжению.)

    • Закон Ома применим только к металлическим проводникам при постоянной температуре.Если температура меняется, закон не действует.
    • Закон Ома также не распространяется на односторонние сети. Обратите внимание, что односторонняя сеть содержит односторонние элементы, такие как транзисторы, диоды и т. Д. Односторонние элементы – это те элементы, которые позволяют току течь только в одном направлении.

    Треугольник закона Ома

    Основные формулы закона Ома подытожены под треугольником закона Ома.

    Треугольник закона Ома

    Круговая диаграмма закона Ома

    Основные формулы закона Ома резюмированы на приведенной ниже круговой диаграмме закона Ома. Круговая диаграмма закона

    Ома

    Проблемы юридической практики Ома

    Пример 1

    Как показано на схеме ниже, через сопротивление 15 Ом протекает ток 4 А. Определите падение напряжения в цепи по закону Ома.

    Решение:

    Данные данные: и

    Согласно закону Ома,

    Таким образом, используя уравнение закона Ома, мы получаем падение напряжения в цепи 60 В.

    Пример 2

    Как показано в приведенной ниже схеме напряжение питания 24 В приложено к сопротивлению 12 Ом.Определите ток, протекающий через резистор, по закону Ома.

    Решение:

    Заданные данные: и

    Согласно закону Ома,

    Таким образом, используя уравнение закона Ома, мы получаем, что ток, протекающий через резистор, равен 2 А.

    Пример 3

    Как Как показано на схеме ниже, напряжение питания составляет 24 В, а ток, протекающий через неизвестное сопротивление, равен 2 А. Определите неизвестное значение сопротивления, используя закон Ома.

    Решение:

    Заданные данные: и

    Согласно закону Ома,

    Таким образом, используя уравнение закона Ома, мы получаем неизвестное значение сопротивления.

    Применение закона Ома

    Некоторые из приложений закона Ома включают:

    • Для вычисления неизвестной разности потенциалов или напряжения, сопротивления и протекания тока в электрической цепи.
    • Закон Ома используется в электронной схеме для определения внутреннего падения напряжения на электронных компонентах.
    • Закон Ома используется в цепях измерения постоянного тока, в частности, в амперметре постоянного тока, в котором для отвода тока используется шунт с низким сопротивлением.

    ОУ, схема компаратора | Renesas

    Введение в электронные схемы: 3 из 3

    На этом занятии мы рассмотрим операционные усилители (операционные усилители) и их использование в усилителях и компараторах.

    Операционные усилители: универсальные ИС для множества приложений

    Операционный усилитель работает на аналоговом входе. Его можно использовать для усиления или ослабления этого входного сигнала, а также для выполнения математических операций, таких как сложение, вычитание, интегрирование и дифференцирование. Из-за широкого диапазона применения операционные усилители встречаются в большинстве электрических цепей.

    Типичный операционный усилитель, показанный на рисунке 1, оборудован неинвертирующим входом (Vin (+)), инвертирующим входом (Vin (-)) и выходом (Vout). Хотя это не показано на схеме, операционный усилитель также имеет два входа питания (положительный и отрицательный), а также может включать в себя вход смещения и другие клеммы.

    Рисунок 1: Схема операционного усилителя

    Основная функция операционного усилителя заключается в значительном усилении разницы между двумя входами и выходе результата. Если вход на V (+) больше, чем на V (-), операционный усилитель будет усиливать и выводить положительный сигнал; если V (-) больше, операционный усилитель будет выдавать усиленный отрицательный сигнал. Двумя другими характеристиками типичного операционного усилителя являются: (а) чрезвычайно высокое входное сопротивление и (б) чрезвычайно низкое выходное сопротивление.

    Поскольку коэффициент усиления операционного усилителя настолько велик, даже небольшие различия на входах быстро приведут выходное напряжение к максимальному или минимальному значению. По этой причине операционные усилители обычно подключаются к отрицательной обратной связи. Давайте посмотрим на пример.

    Основы операционного усилителя (1): Схема инвертирующего усилителя

    Схема, показанная на рис. 2, усиливает и инвертирует (меняет фазу) входной сигнал и выводит результат. В схеме используется отрицательная обратная связь: часть выходного сигнала инвертируется и возвращается на вход.В этом примере обратная связь возникает из-за того, что выход Vout подключен через резистор R2 к инвертирующему входу (-).

    Давайте посмотрим, как работает эта схема. Если выход не подключен к напряжению питания, тогда напряжения, приложенные к инвертирующему (-) и неинвертирующему (+) входам, равны; два входа действуют так, как будто закорочены вместе; мы можем представить себе воображаемую короткую. Поскольку разница напряжений между этим воображаемым коротким замыканием и неинвертирующим входом равна 0 В, точка A также будет равна 0 В.Таким образом, по закону Ома I 1 = Vin / R 1 .

    Рисунок 2: Схема инвертирующего усилителя

    Поскольку операционные усилители имеют чрезвычайно высокий входной импеданс, ток на инвертирующий вход практически отсутствует (-). Соответственно, I 1 протекает через точку A и R 2 ; это означает, что I 1 и I 2 практически равны. Тогда по закону Ома Vout = −I 1 × R 2 , где I 1 отрицательно, потому что I 2 течет из точки A, где напряжение равно 0.Посмотрим на это с другой стороны: любая попытка поднять входное напряжение на инвертирующем входе (-) создает инвертированное и сильно усиленное выходное напряжение, которое течет в обратном направлении, проходит через R 2 и подключается к инвертированной входной клемме (-), тем самым подавление повышения напряжения на этой клемме. Система стабилизируется при выходном напряжении, которое доводит напряжение на инвертирующем входе (-) до 0 В, что эквивалентно напряжению на неинвертирующем входе.

    Далее, давайте посмотрим, как мы можем использовать взаимосвязь между входом и выходом, чтобы найти коэффициент усиления операционного усилителя.В частности, Vout / Vin = (−I 1 × R 2 ) / (I 1 × R 1 ) = −R 2 / R 1 . Коэффициент усиления отрицательный, потому что фаза выходного сигнала противоположна фазе входного сигнала.

    Важно отметить, что в приведенном выше уравнении коэффициент усиления полностью определяется соотношением сопротивлений R 2 и R 1 . Соответственно, вы можете изменить усиление, просто изменив сопротивления. Таким образом, хотя сам операционный усилитель имеет высокое усиление, надлежащее использование отрицательной обратной связи может снизить фактическое усиление до желаемого уровня.

    Основы операционного усилителя (2): Схема неинвертирующего усилителя

    В предыдущем разделе мы видели, как операционный усилитель можно использовать для реализации инвертирующего усилителя. На рисунке 3 показано, как мы можем использовать его для создания неинвертирующего усилителя. Неинвертирующий усилитель отличается от инвертирующего усилителя двумя основными способами: (1) форма выходного сигнала находится в фазе с формой входного сигнала, и (2) входной сигнал поступает на неинвертирующий входной терминал (+). Но обратите внимание, что как неинвертирующие, так и инвертирующие схемы используют отрицательную обратную связь.

    Так как же работает эта схема? У нас все еще есть воображаемое короткое замыкание, что означает, что неинвертирующий (+) и инвертирующий (-) входы находятся под напряжением Vin. Таким образом, точка A также находится в Vin. Закон Ома говорит нам, что напряжение на R 1 равно Vin = R 1 × I 1 . А поскольку на любой из входов операционного усилителя по существу нет тока, отсюда следует, что I 1 = I 2 . А поскольку Vout – это сумма напряжений при R 1 и R 2 , мы знаем, что Vout = R 2 × I 2 + R 1 × I 1 .Мы можем изменить эти выражения, чтобы найти коэффициент усиления G следующим образом: G = Vout / Vin = (1 + R 2 / R 1 )

    Рисунок 3: Схема неинвертирующего усилителя

    Поскольку этот усилитель сохраняет фазу, его часто можно встретить в приложениях, где важно учитывать фазу.

    Также обратите внимание, что если R 1 удален из схемы, а R 2 установлен на 0 Ом (или закорочен), схема становится повторителем напряжения с коэффициентом усиления 1.Этот тип схемы часто используется в схемах буферизации и схемах преобразования импеданса.

    Цепь компаратора

    Схема компаратора сравнивает два напряжения и выдает либо 1 (напряжение на положительной стороне; VDD на иллюстрации), либо 0 (напряжение на отрицательной стороне), чтобы указать, какое из них больше. Компараторы часто используются, например, для проверки того, достиг ли вход некоторого заранее определенного значения. В большинстве случаев компаратор реализуется с использованием специальной микросхемы компаратора, но в качестве альтернативы можно использовать операционные усилители.На схемах компараторов и схемах операционных усилителей используются одни и те же символы.

    На рисунке 4 показана схема компаратора. Прежде всего обратите внимание, что схема не использует обратную связь. Схема усиливает разницу напряжений между Vin и VREF и выводит результат на Vout. Если Vin больше, чем VREF, то напряжение на Vout поднимется до положительного уровня насыщения; то есть к напряжению на положительной стороне. Если Vin ниже VREF, то Vout упадет до своего отрицательного уровня насыщения, равного напряжению на отрицательной стороне.

    На практике эту схему можно улучшить, включив диапазон напряжения гистерезиса, чтобы снизить ее чувствительность к шумам. Например, схема, показанная на рис. 5, будет обеспечивать стабильную работу, даже когда сигнал Vin несколько зашумлен.

    Рисунок 4: Схема компаратора

    Рисунок 5: Схема компаратора с гистерезисом

    Цепь генератора с положительной обратной связью

    Обратная связь – это возврат части выхода схемы обратно на вход схемы с целью некоторого регулирования схемы.При отрицательной обратной связи более высокая обратная связь снижает выходной сигнал схемы. При положительной обратной связи, как в примере здесь, более высокий выход увеличивает выход. Когда положительная обратная связь включена в схему с положительным усилением, схема становится генератором.

    Существует множество типов схем генератора. На рисунке 6 показан пример нестабильного мультивибраторного генератора.

    Рисунок 6: Схема нестабильного мультивибратора

    Эта цепь называется нестабильной, потому что она нестабильна при обоих максимальных напряжениях, напряжении V L на положительной стороне и -V L на отрицательной стороне, и будет колебаться между этими двумя уровнями.Давайте посмотрим, как работает эта схема. Во-первых, обратите внимание, что выходной сигнал Vout проходит через резистор R 2 и обратно на неинвертирующий вывод операционного усилителя (+), образуя цепь положительной обратной связи. Отметим также, что Vout, R 3 и C содержат схему интегратора RC; или, другими словами, некоторая часть напряжения на Vout будет постепенно заряжать конденсатор.

    Вначале цепь обратной связи быстро приводит Vout к максимальному положительному выходу (равному V L ).Но схема интегратора R3 (R 3 и C) постепенно увеличивает напряжение на инвертирующей входной клемме (-), пока через определенное время это напряжение не станет выше, чем напряжение на неинвертирующей входной клемме (+). Когда это происходит, отрицательное напряжение поступает на дифференциальный вход, быстро понижая Vout до максимума на отрицательной стороне (-V L ).

    Теперь, когда Vout находится на отрицательной стороне, схема интегратора R 3 начинает постепенно повышать отрицательное напряжение на инвертирующей клемме (-).И снова, через определенное время, это отрицательное напряжение становится больше, чем напряжение на неинвертирующем выводе (+), вызывая ввод положительного напряжения на дифференциальный вход, который быстро подталкивает Vout обратно к его положительному максимуму ( V L ). Эта последовательность продолжает повторяться, заставляя Vout колебаться вверх и вниз между V L и – V L .

    Оставить комментарий